You are on page 1of 47

IMASEN PHILIPPINE MANUFACTURING CORP. vs.

ALCON
G.R. No. 194884 October 22, 2014
Petitioner Imasen Philippine Manufacturing Corporation is a domestic corporation
engaged in the manufacture of auto seat-recliners and slide-adjusters. It hired the
respondents as manual welders in 2001.
On October 5, 2002, the respondents reported for work on the second shift – from 8:00
pm to 5:00 am of the following day. At around 12:40 am, Cyrus A. Altiche, Imasen’s
security guard on duty, went to patrol and inspect the production plant’s premises.
When Altiche reached Imasen’s Press Area, he heard the sound of a running industrial
fan. Intending to turn the fan off, he followed the sound that led him to the plant’s “Tool
and Die” section.
At the “Tool and Die” section, Altiche saw the respondents having sexual intercourse on
the floor, using a piece of carton as mattress. Altiche immediately went back to the
guard house and relayed what he saw to Danilo S. Ogana, another security guard on
duty. On Altiche’s request, Ogana madea follow-up inspection. Ogana went to the “Tool
and Die” section and saw several employees, including the respondents, already leaving
the area. He noticed, however, that Alcon picked up the carton that Altiche claimed the
respondents used as a mattress during their sexual act, and returned it to the place
where the cartons were kept. Altiche then submitted a handwritten report of the
incident to Imasen’s Finance and Administration Manager.
On October 14, 2002, Imasen issued the respondents separate interoffice memoranda
informing them of Altiche’s report on the October 5, 2002 incident and directing them
to submit their individual explanation. The respondents complied with the directive;
they claimed that they were merely sleeping in the “Tool and Die” section at the time of
the incident. They also claimed that other employees were near the area, making the
commission of the act charged impossible.
On December 4, 2002, Imasen issued the respondents separate interoffice memoranda
terminating their services. It found the respondents guilty of the act charged which it
considered as “gross misconduct contrary to the existing policies, rules and regulations
of the company.”
On December 5, 2002, the respondents filed before the LA theComplaint for illegal
dismissal. The respondents maintained their version of the incident.The LA dismissed
the respondents’ complaint for lack of merit. The LA found the respondents’ dismissal
valid, i.e., for the just cause of gross misconduct and with due process. Such ruling was
affirmed by the NLRC, but was reversed by the CA. To the CA, the penalty of dismissal is
not commensurate to the respondents’ act, considering especially that the respondents
had not committed any infraction in the past.
Petitioner’s Contention
Imasen argues in this petition that the act of engaging in sexual intercourse inside
company premises during work hours is serious misconduct by whatever standard it is
measured. According to Imasen, the respondents’ infraction is an affront to its core
values and high ethical work standards, and justifies the dismissal.
Respondents’ Contention
The respondents argue in their comment that the elements of serious misconduct that
justifies an employee’s dismissal are absent in this case, adopting thereby the CA’s
ruling.
ISSUE:
Whether or not sexual intercourse inside the company premises is a ground for
dismissal of service.
RULING:
Yes. The just causes for dismissing an employee are provided under Article 282 (now
Article 296) of the Labor Code. Under Article 282(a), serious misconduct by the
employee justifies the employer in terminating his or her employment.
Misconduct is defined as an improper or wrong conduct. It is a transgression of some
established and definite rule of action, a forbidden act, a dereliction of duty, willful in
character, and implies wrongful intent and not mere error in judgment.
To constitute a valid cause for the dismissal within the text and meaning of Article 282
of the Labor Code, the employee’s misconduct must be serious, i.e., of such grave and
aggravated character and not merely trivial or unimportant.
For misconduct or improper behavior to be a just cause for dismissal, the following
elements must concur: (a) the misconduct must be serious; (b) it must relate to the
performance of the employee’s duties showing that the employee has become unfit to
continue working for the employer; and (c) it must have been performed with wrongful
intent.
The respondents’ infraction amounts to serious misconduct within the terms of Article
282 (now Article296) of the Labor Code justifying their dismissal.
Dismissal situations (on the ground of serious misconduct) involving sexual acts,
particularly sexual intercourse committed by employees inside company premises and
during work hours, are not usual violations and are not found in abundance under
jurisprudence. Thus, in resolving the present petition, we are largely guided by the
principles we discussed above, as applied to the totality of the circumstances that
surrounded the petitioners’ dismissal.
Sexual acts and intimacies between two consenting adults belong, as a principled ideal,
to the realm of purely private relations. Whether aroused by lust or inflamed by sincere
affection, sexual acts should be carried out at such place, time and circumstance that, by
the generally accepted norms of conduct, will not offend public decency nor disturb the
generally held or accepted social morals. Under these parameters, sexual acts between
two consenting adults do not have a place in the work environment.
Indisputably, the respondents engaged in sexual intercourse inside company premises
and during work hours. These circumstances, by themselves, are already punishable
misconduct. Added to these considerations, however, is the implication that the
respondents did not only disregard company rules but flaunted their disregard in a
manner that could reflect adversely on the status of ethics and morality in the company.
Additionally, the respondents engaged in sexual intercourse in an area where co-
employees or other company personnel have ready and available access. The
respondents likewise committed their act at a time when the employees were expected
to be and had, in fact, been at their respective posts, and when they themselves were
supposed to be, as all other employees had in fact been, working.
Under these factual premises and in the context of legal parameters we discussed, we
cannot help but consider the respondents’ misconduct to be of grave and aggravated
character so that the company was justified in imposing the highest penalty available ―
dismissal. Their infraction transgressed the bounds of socially and morally accepted
human public behavior, and at the same time showed brazen disregard for the respect
that their employer expected of them as employees. By their misconduct, the
respondents, in effect, issued an open invitation for others to commit the same
infraction, with like disregard for their employer’s rules, for the respect owed to their
employer, and for their co-employees’ sensitivities. Taken together, these considerations
reveal a depraved disposition that the Court cannot but consider as a valid cause for
dismissal.
All told, the respondents’ misconduct, under the circumstances of this case, fell within
the terms of Article 282 (now Article 296) of the Labor Code. Consequently, we reverse
the CA’s decision for its failure to recognize that no grave abuse of discretion attended
the NLRC’s decision to support the respondents’ dismissal for serious misconduct.
 
NORTHWEST AIRLINES, INC. v. MA. CONCEPCION M. DEL ROSARIO;
G.R. No. 157633, September 10, 2014
Northwest Airlines, Inc. employed respondent Ma. Concepcion M. Del Rosario (Del
Rosario) on December 10, 1994 as one of its Manila-based flight attendants. On May 18,
1998, both Del Rosario and Kathleen Gamboa (Gamboa) another flight attendant was
assigned at Northwest Flight NW 26 bound for Japan, the former was assigned at the
Business Class Section, while the latter, at the First Class Section. Gamboa ask her
runner Vivien Francisco (Francisco) to borrow a wine bottle opener because her wine
bottle opener was dull. Francisco went to the Business Class Section and ask Del
Rosario, but the latter remarked that any flight attendant who could not bring a wine
bottle opener had no business working in the First Class Section. Upon hearing this,
Aliza Ann Escaño (Escaño), another flight attendant, offered her wine bottle opener to
Francisco. Apparently, Gamboa overheard Del Rosario’s remarks, and later on verbally
confronted her. Their confrontation escalated into a heated argument. Escaño
intervened but the two ignored her, prompting her to rush outside the aircraft to get
Maria Rosario D. Morales (Morales), the Assistant Base Manager, to pacify them.
Morales asked them about what had happened, and even asked if they were willing to fly
on the condition that they would have to stay away from each other during the entire
flight; that because Del Rosario was not willing to commit herself to do so, she decided
not to allow both of them on Flight NW 26, and furnished them a Notice of Removal
from Service (effectively informing Del Rosario of her dismissal from the service
pending an investigation of the fighting incident between her and Gamboa).
On May 19, 1998, Morales sent a letter to Del Rosario telling her that Northwest would
conduct an investigation of the incident involving her and Gamboa. The investigation
was held on May 28, 1998 before Atty. Ceazar Veneracion III, Northwest’s Legal Counsel
and Head of its Human Resources Department. All the parties attended the
investigation. On June 19, 1998, Del Rosario was informed of her termination from the
service. Del Rosario subsequently filed her complaint for illegal dismissal against
Northwest.
Issue/s:
1. Whether or not Del Rosario’s dismissal from the service valid?
Ruling:
No. There is no doubt that the last two elements of misconduct were present in the case
of Del Rosario. The cause of her dismissal related to the performance of her duties as a
flight attendant, and she became unfit to continue working for Northwest. However,
misconduct was not as serious as to merit Del Rosario’s dismissal under Article 297 of
the Labor Code.
Labor Arbiter Teresita D. Castillon-Lora ruled in favor of Northwest, holding that the
dismissal of Del Rosario had been justified and valid upon taking into account that
Northwest had been engaged in the airline business in which a good public image had
been demanded, and in which flight attendants had been expected to maintain an image
of sweetness and amiability; that fighting among its employees even in the form of
heated arguments or discussions were very contradictory to that expected image; and
that it could validly dismiss its employees like the respondent because it had been
entitled to protect its business interests by putting up an impeccable image to the public.
NLRC reversed the decision of the Labor Arbiter, and ruled in favor of Del Rosario,
declaring that the incident between her and Gamboa could not be considered as
synonymous with fighting as the activity prohibited by Northwest’s Rules of Conduct;
that based on Black’s Law Dictionary, fight referred to a hostile encounter, affray, or
altercation; a physical or verbal struggle for victory, pugilistic combat; that according to
Bouvier’s Law Dictionary, fighting did not necessarily imply that both parties should
exchange blows, for it was sufficient that they voluntarily put their bodies in position
with that intent; and that the incident between Del Rosario and Gamboa could not be
held similar to the fight that Northwest penalized under its Rules of Conduct. The NLRC
ordered the reinstatement of Del Rosario to her former position without loss of seniority
rights and with payment of backwages, per diems, other lost income and benefits from
June 19, 1998; as well as the payment of attorney’s fees equivalent to 10% of the
monetary award.
CA sustained the NLRC through its decision promulgated on June 21, 2002, observing
that Northwest did not discharge its burden to prove not merely reversible error but
grave abuse of discretion amounting to lack or excess of jurisdiction on the part of the
NLRC; and that, indeed, the NLRC had correctly held that Del Rosario’s conduct did not
constitute serious misconduct, because the NLRC, in determining the usual, ordinary
and commonly understood meaning of the word fighting, had resorted to authoritative
lexicons that supported its conclusion that the exchange of words between Del Rosario
and Gamboa did not come within the definition of the word fighting.
CITIBANK N.A. vs. NLRC and ROSITA TAN PARAGAS
G.R. 159302 February 6, 2008
Rosita Tan Paragas (Paragas) worked as a filing clerk of Citibank, N.A. (Citibank) for
eighteen (18) years. Sometime in the early part of 1993, as a result of reorganization,
respondent bank declared certain officers and employees, or their positions/functions,
redundant. Among these affected was complainant Paragas. However, complainant’s
employment was not terminated but was assigned to Records Management Unit of the
Quality Assurance Division as bank statement retriever, a filing clerk job described by
complainant as "non-brainer job."
In the latter part of July 1994, complainant was assigned to file Universal Account
Opening Forms (UAOF) in file boxes and to retrieve such UAOFs from the file boxes
upon internal customers’ request from time to time. In the same month, she was also
assigned to process or develop microfilms. However, on 20 February 1995, she
complained that the processing of microfilms was proving to be harmful to her health.
Thus, the job was reassigned to another clerk. Accordingly, beginning 21 February 1995,
complainant’s job in the bank was to file and retrieve UAOFs.
On 11 December 1996, complainant was assigned to undertake the special project of
reorganizing the UAOF’s from 13 December 1996 to 15 May 1997. On 10 January 1997,
AVP Narciso Ferrera issued a Memo to complainant calling her attention on the
following: (a) various misfiling on the reorganized UAOF files; (b) continuous failure to
submit weekly status reports; (c) trimming/cutting of edges of attached documents; and
(d) accumulation of incoming newly received UAOFs.
Again, on 2 April 1997, complainant received another memo from AVP Ferrera which
called her attention (a) to the same nine (9) cases misfiled UAOF’s in Annex 16, (b) to
three (3) other cases of misfiled UAOFs (c) her persistent failure to submit weekly report
on the progress of her work under the Special Project, and (d) that despite the lapse of
three (3) months, she was still in letter D (or UAOFs covering clients whose surnames
begin with letter D).
As she failed to complete the project on 30 May 1997, complainant was given another 30
days to complete it. However, by the end of June 1997, her accomplishment was only
30% of the total work to be done.
On 25 July 1997, AVP Ferrera directed complainant to explain in writing why her
employment should not be terminated on the ground of serious misconduct, willful
disobedience, gross and habitual neglect of her duties and gross inefficiency.
Correspondingly, complainant was placed under Preventive suspension. Complainant
submitted her written explanation on 31 July 1997.
On 29 August 1997, an administrative conference took place with the complainant, her
counsel and the Union President in attendance.
Finally, on 4 September 1997, the respondent bank thru AVP Ferrera notified
complainant that her written explanation and those which she ventilated during the
administrative conference held on 29 August 1997 were found self-serving, and
consequently, terminating her employment on the ground of serious misconduct, willful
disobedience, gross and habitual neglect of duties and gross inefficiency.
Paragas filed a complaint for illegal dismissal which was dismissed for lack of merit,
finding that the dismissal on the ground of work inefficiency was valid. The NLRC
affirmed the decision of the Labor Arbiter with the modification that Paragas should be
paid separation pay as a form of equitable relief in view of her length of service with
Citibank.
Paragas filed a Motion for Partial Reconsideration of the NLRC Resolution. She no
longer challenged her dismissal on the ground of work inefficiency, but prayed that
Citibank be ordered to pay her the Provident Fund benefits under its retirement plan for
which she claimed to be qualified pursuant to Citibank’s Working Together Manual. The
said manual provides that an employee discharged for reasons other than misconduct
will be paid a percentage of her share in the Fund.
Finding that Paragas’ dismissal was for causes other than misconduct, the NLRC
granted Paragas’ Motion. On appeal, the Court of Appeals dismissed the petition for lack
of merit and affirmed in toto the challenged NLRC Resolution.
ISSUE:
Whether or not Paragas is entitled to retirement benefits.
HELD: No.
Paragas indeed prayed for “other just and equitable relief,” but the same may not be
interpreted so broadly as to include even those which are not warranted by the factual
premises alleged by a party. Thus the January 24, 2003 Decision of the Court of Appeals
correctly stated: “It has been ruled in this jurisdiction that the general prayer for ‘other
reliefs’ is applicable to such other reliefs which are warranted by the law and facts
alleged by the respondent in her basic pleadings and not on a newly created issue.” Note
that Paragas filed her "Notice of Appeal and Appeal Memorandum" after she had
already submitted her position paper.
Paragas justifies her failure to claim for retirement benefits before the Labor Arbiter by
alleging that it would be inconsistent with her prayer for reinstatement. Paragas,
however, could have easily claimed such benefits as an alternative relief.
In any event, Paragas is not entitled to retirement benefits as this Court finds that she
was validly dismissed for serious misconduct and not merely for work
inefficiency.
In support of its ruling that respondent’s dismissal was valid, the Labor Arbiter relied on
the performance appraisals of respondent from July to December 1994, from January to
June 1995, and from July to December 1996, all of which were submitted by petitioner’s
Assistant Vice-President, Narciso M. Ferrera.
These performance appraisals, however, did not merely show that respondent was not
able to meet performance targets. More relevantly, they also consistently noted
significant behavioral and attitudinal problems in respondent. In particular, respondent
was found to be very argumentative; she had difficulty working with others;
she was hard to deal with; and she never ceased being the subject of
complaints from co-workers.
Moreover, beyond the documents referred to in the labor arbiter’s decision, there are
other pieces of evidence on record which further establish that respondent was validly
dismissed not only for work inefficiency but for serious misconduct.
The assessment of respondent’s performance by Randy Uson, another superior of
respondent, was given weight by the Labor Arbiter. Significantly, Uson later commented
on respondent’s behavior as follows:

"Less tangible but none the less real, are the common concerns raised by her peers and
supervisor, on the stress and tension created when Rose is around. The conscious
effort to ‘get out of her way’ and avoid conflict, hinders productivity and efficiency and
has adversely affected the morale of the entire unit. x x x”

Furthermore, for the appraisal period from June to December 1995, respondent’s
performance appraisal report stated that her attitude towards her work, the bank, and
superiors needed reformation. The report for January to June 1996 made the same
observation, indicating that there was no improvement on her part.
The performance appraisal report of respondent for the period of January to June 1997,
besides stating that she was still "hard to deal with," described her as "belligerent," one
who had "a negative presence which affects the morale of the entire unit," and who
"pick[ed] fights with peers and other employees even without provocation.”
The evaluation of respondent cited above finds corroboration in her admission that "she
may have been tactless and insolent in dealing with her superior but it does not allegedly
warrant the supreme penalty of dismissal.”
Finally, even the NLRC, its later ruling that respondent was not guilty of misconduct
notwithstanding, was aware that the problem with respondent was not merely her poor
work output, but her unreasonable behavior and unpleasant deportment.
When an employee, despite repeated warnings from the employer,
obstinately refuses to curtail a bellicose inclination such that it erodes the
morale of co-employees, the same may be a ground for dismissal for serious
misconduct.
As this Court held in National Service Corp. v. Leogardo, Jr., "[a] series of irregularities
when put together may constitute serious misconduct, which under Article 283 of the
Labor Code, is a just cause for termination." And as it held in Asian Design and
Manufacturing Corporation v. Deputy Minister of Labor, acts destructive of the morale
of one’s co-employees may be considered serious misconduct.
It is respondent’s obstinate refusal to reform herself which ultimately persuades this
Court to find that her dismissal on the ground of serious misconduct was valid.
Having been validly dismissed on the ground of serious misconduct, respondent is thus
disqualified from receiving her retirement benefits pursuant to the provision of
petitioner’s "Working Together" Manual quoted earlier.

MIRANT (PHILIPPINES) CORP. vs. JOSELITO A. CARO


G.R. No. 181490 : April 23, 2014

Petitioner corporation is organized and operating under and by virtue of the laws of the
Republic of the Philippines. It is a holding company that owns shares in project
companies such as Mirant Sual Corporation and Mirant Pagbilao Corporation (Mirant
Pagbilao) which operate and maintain power stations located in Sual, Pangasinan and
Pagbilao, Quezon, respectively. Petitioner corporation and its related companies
maintain around 2,000 employees detailed in its main office and other sites. Petitioner
corporation had changed its name to CEPA Operations in 1996 and to Southern
Company in 2001. In 2002, Southern Company was sold to petitioner Mirant whose
corporate parent is an Atlanta-based power producer in the United States of America.
Petitioner corporation is now known as Team Energy Corporation.

Petitioner Edgardo A. Bautista (Bautista) was the President of petitioner corporation


when respondent was terminated from employment.

Respondent was hired by Mirant Pagbilao on January 3, 1994 as its Logistics Officer. In
2002, when Southern Company was sold to Mirant, respondent was already a
Supervisor of the Logistics and Purchasing Department of petitioner. At the time of the
severance of his employment, respondent was the Procurement Supervisor of Mirant
Pagbilao assigned at petitioner corporations corporate office. As Procurement
Supervisor, his main task was to serve as the link between the Materials Management
Department of petitioner corporation and its staff, and the suppliers and service
contractors in order to ensure that procurement is carried out in conformity with set
policies, procedures and practices. In addition, respondent was put in charge of
ensuring the timely, economical, safe and expeditious delivery of materials at the right
quality and quantity to petitioner corporations plant. Respondent was also responsible
for guiding and overseeing the welfare and training needs of the staff of the Materials
Management Department. Due to the nature of respondents functions, petitioner
corporation considers his position as confidential.
Respondent filed a complaint for illegal dismissal and money claims for 13th and 14th
month pay, bonuses and other benefits, as well as the payment of moral and exemplary
damages and attorneys fees. It is the contention of respondent that he was illegally
dismissed by petitioner corporation due to the latters non-compliance with the twin
requirements of notice and hearing. He asserts that while there was a notice charging
him of unjustified refusal to submit to random drug testing, there was no notice of
hearing and petitioner corporations investigation was not the equivalent of the hearing
required under the law which should have accorded respondent the opportunity to be
heard.

In a decision dated August 31, 2005, Labor Arbiter Aliman D. Mangandog found
respondent to have been illegally dismissed. The Labor Arbiter also found that the
quitclaim purportedly executed by respondent was not a bona fide quitclaim which
effectively discharged petitioners of all the claims of respondent in the case at bar. If at
all, the Labor Arbiter considered the execution of the quitclaim as a clear attempt on the
part of petitioners to mislead its office into thinking that respondent no longer had any
cause of action against petitioner corporation.

On appeal to the NLRC, petitioners alleged that the decision of the Labor Arbiter was
rendered with grave abuse of discretion for being contrary to law, rules and established
jurisprudence, and contained serious errors in the findings of facts which, if not
corrected, would cause grave and irreparable damage or injury to petitioners. The
NLRC, giving weight and emphasis to the inconsistencies in respondents explanations,
considered his omission as unjustified refusal in violation of petitioner corporations
drug policy. Respondent filed a motion for reconsideration, while petitioners filed a
motion for partial reconsideration of the NLRC decision. In a Resolution dated June 30,
2006, the NLRC denied both motions.

ISSUE: Whether respondent was illegally dismissed

HELD:
We agree with the disposition of the appellate court that there was illegal
dismissal in the case at bar.
While the adoption and enforcement by petitioner corporation of its Anti-Drugs
Policy is recognized as a valid exercise of its management prerogative as an
employer, such exercise is not absolute and unbridled. Managerial prerogatives
are subject to limitations provided by law, collective bargaining agreements, and
the general principles of fair play and justice. In the exercise of its management
prerogative, an employer must therefore ensure that the policies, rules and
regulations on work-related activities of the employees must always be fair and
reasonable and the corresponding penalties, when prescribed, commensurate to
the offense involved and to the degree of the infraction. The Anti-Drugs Policy of
Mirant fell short of these requirements.

Petitioner corporations subject Anti-Drugs Policy fell short of being fair and
reasonable.

First. The policy was not clear on what constitutes unjustified refusal when the
subject drug policy prescribed that an employees unjustified refusal to submit to
a random drug test shall be punishable by the penalty of termination for the first
offense. To be sure, the term unjustified refusal could not possibly cover all forms
of refusal as the employees resistance, to be punishable by termination, must be
unjustified. To the mind of the Court, it is on this area where petitioner
corporation had fallen short of making it clear to its employees as well as to
management as to what types of acts would fall under the purview of unjustified
refusal. Even petitioner corporations own Investigating Panel recognized this
ambiguity.

JEFFREY NACAGUE VS. SULPICIO LINES, INC.


G.R. No. 172589. August 08, 2010

On 15 June 1995, respondent Sulpicio Lines, Inc. (Sulpicio Lines) hired Nacague as
"hepe de viaje" or the representative of Sulpicio Lines on board its vessel M/V Princess
of the World (the ship).
 
On 15 February 2003, Sulpicio Lines sent a notice of investigation to Nacague informing
him of the charges against him for use of illegal drugs and threatening a co-employee.[9]
 
When the ship docked in the port of Manila on 18 February 2003, some crew members
of the ship, together with Nacague, were subjected to a random drug test. They were
taken to S.M. Lazo Medical Clinic (S.M. Lazo Clinic) and were required to submit urine
samples. The result of the random drug test revealed that Nacague was positive for
methamphetamine hydrochloride or shabu.[10]
 
On 20 February 2003, Sulpicio Lines subjected Nacague to a formal investigation.
Nacague denied using illegal drugs.[11]
 
On 23 February 2003, Nacague went to Chong Hua Hospital in Cebu City to undergo a
voluntary drug test. The drug test with Chong Hua Hospital yielded a negative result.
[12] Nacague submitted this test result to Sulpicio Lines.
 
However, on 7 March 2003, Sulpicio Lines sent a memorandum to Nacague terminating
him from the service.
 
Feeling aggrieved, Nacague filed a complaint for illegal suspension, illegal dismissal and
for reinstatement with backwages.
 
On 12 November 2003, Labor Arbiter Ernesto F. Carreon rendered a decision in favor of
Nacague and declared that Sulpicio Lines illegally dismissed Nacague. [14] [15]
 
According to the Labor Arbiter, the termination of employment of employees found
positive for using illegal drugs should not be exercised indiscriminately and
thoughtlessly. The Labor Arbiter agreed with Nacague that the drug test result from
S.M. Lazo Clinic was questionable because the clinic is not accredited by the
Dangerous Drug Board and not under its supervision. The Labor Arbiter gave
more weight to the drug test performed by Chong Hua Hospital because it was
accredited by the Dangerous Drug Board. The Labor Arbiter said that doubts must be
resolved in favor of the employee. The Labor Arbiter also ruled that reinstatement is no
longer viable due to the strained relations between Nacague and Sulpicio Lines and,
thus, awarded separation pay to Nacague.
 
Dissatisfied with the Labor Arbiter's Decision, Sulpicio Lines appealed to the NLRC.
 
According to the NLRC, since Nacague, who was performing a task involving trust and
confidence, was found positive for using illegal drugs, he was guilty of serious
misconduct and loss of trust and confidence. The NLRC added that Sulpicio Lines' Code
of Conduct[16] specified that the penalty for the use and illegal possession of prohibited
drugs is dismissal. The NLRC also said that there is a presumption that S.M. Lazo Clinic
is an accredited drug testing center and that it was incumbent upon Nacague to show
otherwise.
 
Nacague filed a motion for reconsideration. In its 31 May 2005 Resolution, the NLRC
denied Nacague's motion.
 
Nacague filed a petition for certiorari with the Court of Appeals. Nacague alleged that
the NLRC gravely abused its discretion when it declared that Sulpicio Lines validly
terminated his employment.
 
The Court of Appeals declared that the evidence presented by Sulpicio Lines was
sufficient to justify the conclusion that Nacague committed serious misconduct and a
breach of trust and confidence warranting his dismissal from employment. The Court of
Appeals agreed with the NLRC that Nacague failed to prove his allegation that S.M. Lazo
Clinic lacks accreditation. On the procedural requirements, the Court of Appeals found
that Sulpicio Lines complied with the twin-notice requirements and conducted a formal
hearing.
 
ISSUE: Nacague raises the sole issue of whether the Court of Appeals erred in ruling
that his termination from employment was valid.
 
HELD: The petition is meritorious. The Supreme Court GRANTED Nacague's
petition.
 
Nacague maintains that the S.M. Lazo Clinic drug test was not credible because Sulpicio
Lines failed to show that S.M. Lazo Clinic is an authorized drug testing center. Nacague
also alleges that the urine samples were gathered carelessly without proper labels to
identify their owners and that S.M. Lazo Clinic did not ask Nacague if he was taking any
medication that might alter the results of the drug test. [17]Nacague adds that Republic
Act No. 9165[18] (R.A. No. 9165) and the Department of Labor and Employment Order
No. 53-03[19] (Department Order No. 53-03) require two drug tests -- a screening test
and a confirmatory test. Nacague maintains that, since only a screening test was
conducted, he was illegally dismissed based on an incomplete drug test. Nacague argues
that Sulpicio Lines failed to discharge its burden of proving that the termination of his
employment was legal.
 
Under Article 279[20] of the Labor Code, an employer may terminate the services of an
employee for just causes[21] or for authorized causes.[22] Furthermore, under Article
277(b)[23] of the Labor Code, the employer must send the employee who is about to be
terminated, a written notice stating the causes for termination and must give the
employee the opportunity to be heard and to defend himself.
 
Contrary to Sulpicio Lines' allegation, Nacague was already questioning the credibility of
S.M. Lazo Clinic as early as the proceedings before the Labor Arbiter. In fact, the Labor
Arbiter declared that the S.M. Lazo Clinic drug test result was doubtful since it is not
under the supervision of the Dangerous Drug Board.[25]
 
Sulpicio Lines failed to clearly show that Nacague was guilty of using illegal
drugs. We agree with the Labor Arbiter that the lack of accreditation of S.M.
Lazo Clinic made its drug test results doubtful.
Section 36 of R.A. No. 9165 provides that drug tests shall be performed only by
authorized drug testing centers. Moreover, Section 36 also prescribes that drug testing
shall consist of both the screening test and the confirmatory test. Section 36 of R.A. No.
9165 reads:
SEC. 36. Authorized Drug Testing. Authorized drug testing shall be done by any
government forensic laboratories or by any of the drug testing laboratories
accredited and monitored by the DOH to safeguard the quality of test
results. 
In Social Justice Society v. Dangerous Drugs Board,[26] we explained:
As to the mechanics of the test, the law specifies that the procedure shall employ two
testing methods, i.e., the screening test and the confirmatory test, doubtless to ensure
as much as possible the trustworthiness of the results. But the more important
consideration lies in the fact that the tests shall be conducted by trained
professionals in access-controlled laboratories monitored by the
Department of Health (DOH) to safeguard against results tampering and to ensure an
accurate chain of custody.
The law is clear that drug tests shall be performed only by authorized drug testing
centers. In this case, Sulpicio Lines failed to prove that S.M. Lazo Clinic is an accredited
drug testing center. Sulpicio Lines did not even deny Nacague's allegation that S.M.
Lazo Clinic was not accredited. Also, only a screening test was conducted to determine if
Nacague was guilty of using illegal drugs. Sulpicio Lines did not confirm the positive
result of the screening test with a confirmatory test. Sulpicio Lines failed to indubitably
prove that Nacague was guilty of using illegal drugs amounting to serious misconduct
and loss of trust and confidence. Sulpicio Lines failed to clearly show that it had a valid
and legal cause for terminating Nacague's employment. When the alleged valid cause for
the termination of employment is not clearly proven, as in this case, the law considers
the matter a case of illegal dismissal.
 
CHERYLL SANTOS LEUS vs. ST. SCHOLASTICA'S COLLEGE
G.R. No. 187226 January 28, 2015
 
Petitioner was hired by St. Scholastica's College Westgrove (SSCW), a Catholic
educational institution, as a non-teaching personnel. She engaged in pre-marital sexual
relations, got pregnant out of wedlock, married the father of her child, and was
dismissed by SSCW, in that order. The question that has to be resolved is whether the
petitioner's conduct constitutes a ground for her dismissal.
SSCW is a Catholic and Sectarian educational institution in Silang, Cavite. In May 2001,
SSCW hired the petitioner as an Assistant to SSCW’s Director of the Lay Apostolate and
Community Outreach Directorate.
Sometime in 2003, petitioner and her boyfriend conceived a child out of wedlock. When
SSCW learned of the petitioner's pregnancy, Sr. Edna Quiambao advised her to file a
resignation letter effective June 1, 2003. In response, the petitioner informed Sr.
Quiambao that she would not resign from her employment just because she got
pregnant without the benefit of marriage.
Quiambao formally directed the petitioner to explain in writing why she should not be
dismissed for engaging in pre-marital sexual relations and getting pregnant as a result
thereof, which amounts to serious misconduct and conduct unbecoming of an employee
of a Catholic school.
In a letter dated June 6, 2003, SSCW, through counsel, maintained that pre-marital
sexual relations, even if between two consenting adults without legal impediment to
marry, is considered a disgraceful and immoral conduct or a serious misconduct, which
are grounds for the termination of employment. Petitioner thus filed a complaint for
illegal dismissal
Issues:
The validity of the petitioner's dismissal hinges on the determination of whether
pregnancy out of wedlock by an employee of a catholic educational institution is a cause
for the termination of her employment.
Ruling:
The fact of the petitioner's pregnancy out of wedlock, without more, is not enough to
characterize the petitioner's conduct as disgraceful or immoral.
There must be substantial evidence to establish that pre-marital sexual relations and,
consequently, pregnancy out of wedlock, are indeed considered disgraceful or immoral.
The totality of the circumstances surrounding the conduct alleged to be disgraceful or
immoral must be assessed against the prevailing norms of conduct.
There must be a consideration of the totality of the circumstances surrounding the
conduct; and second, an assessment of the said circumstances vis-à-vis the prevailing
norms of conduct, i.e., what the society generally considers moral and respectable.
The right of an employee to security of tenure is protected by the Constitution.
When the law refers to morality, it necessarily pertains to public and secular morality
and not religious morality. Thus, the proscription against "disgraceful or immoral
conduct" under Section 94(e) of the 1992 MRPS, which is made as a cause for dismissal,
must necessarily refer to public and secular morality.
 
THE COFFEE BEAN AND TEA LEAF v. ROLLY P. ARENAS
Arenas was employed as barista by CBTL who was in-charged of taking order and and
preparing their ordered food and beverages. To ensure the quality of its crew’s services,
CBTL regularly employs a “mystery guest shopper” who poses as a customer, for the
purpose of covertly inspecting the baristas’ job performance. The mystery guest shopper
and manager imputed the following against Arenas:
1. Leaving the counter unattended and eating chips in an unauthorized area while on
duty (March
30, 2009);
2. Reporting late for work on several occasions (April 1, 3 and 22); and
3. Placing an iced tea bottle in the ice bin despite having knowledge of company policy
prohibiting the same (April 28, 2009).
By these reasons he was terminated from work and consequently filed for illegal
dismissal.
LA & NLRC- rule that CBTL guilty of illegal dismissal
CA- reversed
ISSUE: WON Arenas committed serious misconduct or willful disobedience, and gross
negligence, to merit his termination from service
HELD:
NO.
Willful disobedience to be a valid cause for dismissal, these two elements must concur:
(1) the employee’s assailed conduct must have been willful, that is, characterized by a
wrongful and perverse attitude; and (2) the order violated must have been reasonable,
lawful, made known to the employee, and must pertain to the duties which he had been
engaged to discharge.
In this case, it is clear that Arenas’ alleged infractions do not amount to such a wrongful
and perverse attitude. Though Arenas may have admitted these wrongdoings, these do
not amount to a wanton disregard of CBTL’s company policies. As Arenas mentioned in
his written explanation, he was on a scheduled break when he was caught eating at
CBTL’s al fresco dining area. During that time, the other service crews were the one in
charge of manning the counter.
Gross negligence implies a want or absence of, or failure to exercise even a slight care or
diligence, or the entire absence of care. The three counts of tardiness cannot be
considered as gross and habitual neglect of duty. The infrequency of his tardiness
already removes the character of habitualness.
For misconduct or improper behavior to be a just cause for dismissal, (a) it must be
serious; (b) it must relate to the performance of the employee’s duties; and (c) it must
show that the employee has become unfit to continue working for the employer. The
facts on record reveal that there was no active dishonesty on the part of Arenas. When
questioned about who placed the bottled iced tea inside the ice bin, his immediate
reaction was not to deny his mistake, but to remove the bottle inside the bin and throw
it outside. More importantly, when he was asked to make a written explanation of his
action, he admitted that the bottled iced tea was his.
 
LORES REALTY ENTERPRISES, INC. vs. VIRGINIA E. PACIA
G.R. No. 171189 : March 9, 2011
Respondent Virginia E. Pacia(Pacia)was hired by Lores Realty Enterprises, Inc. (LREI).
At the time of her dismissal, she was the assistant manager and officer-in-charge of
LREI's Accounting Department under the Finance Administrative Division.
LREI's acting general manager, Sumulong, ordered Pacia to prepare a check amounting
to P150,000.00, an order which she was slow to follow. Pacia eventually complied with
the order, nevertheless. Pacia was ordered to prepare another check, this time
amounting to P175,000.00. She was again slow to comply, but eventually the order was
complied with. To explain her refusal to immediately follow the directive, Pacia
reasoned out that the funds in LREIs account were not sufficient to cover the amounts
to be indicated in the checks.
Thereafter Pacia received a notice of termination stating, among others, that she was
being dismissed because of her willful disobedience and their loss of trust and
confidence in her.
Pacia then filed a Complaint for Unfair Labor Practice due to Harassment, Constructive
Dismissal, Moral and Exemplary Damages against LREI and Sumulong. Subsequently,
Pacia filed an Amended Complaint to include the charges of illegal dismissal and non-
payment of salaries. The Labor Arbiter(LA)rendered a decision finding that the
dismissal of Pacia was for a just and valid cause but ordering payment of what was due
her.On appeal, the NLRC in its decision reversed the LA's Decision and found LREI and
Sumulong guilty of illegal dismissal. The case was elevated to the CA. The CA held that
LREI and Sumulong failed to establish with substantial evidence that the dismissal of
Pacia was for a just cause.It found that Pacias initial reluctance to obey the orders of her
superiors was for a good reason - to shield the company from liability in the event that
the checks would be dishonored for insufficiency of funds.
ISSUE:
Whether or not Pacias termination was justified under the circumstances
HELD: The petition has no merit.
At the outset, it must be emphasized that the issues raised in this petition are questions
of fact which are not proper subjects of an appeal by certiorari.A disharmony between
the factual findings of the LA and the NLRC, however,opens the door to a review by this
Court.Factual findings of administrative agencies are not infallible and will be set aside
when they fail the test of arbitrariness. Moreover, when the findings of the NLRC
contradict those of the LA, this Court, in the exercise of its equity jurisdiction, may look
into the records of the case and re-examine the questioned findings.
The offense of willful disobedience requires the concurrence of two (2) requisites:
(1) the employee's assailed conduct must have been willful, that is characterized by a
wrongful and perverse attitude; and
(2) the order violated must have been reasonable, lawful, made known to the employee
and must pertain to the duties which he had been engaged to discharge.
The Court finds nothing unlawful in the directive of Sumulong to prepare checks in
payment of LREI's obligations. The availability or unavailability of sufficient funds to
cover the check is immaterial in the physical preparation of the checks. Pacias initial
reluctance to prepare the checks, however, which was seemingly an act of disrespect and
defiance, was for honest and well intentioned reasons. Protecting LREI and Sumulong
from liability under the Bouncing Checks Law. was foremost in her mind.It was not
wrongful or willful. Neither can it be considered an obstinate defiance of company
authority.The Court takes into consideration that Pacia, despite her initial reluctance,
eventually did prepare the checks on the same day she was tasked to do it. Pacias
apprehension was justified when the check was dishonored.This clearly affirms her
assertion that she was just being cautious and circumspect for the company's sake.Thus,
her actuation should not be construed as improper conduct.
ePACIFIC GLOBAL CONTACT CENTER, INC. vs. MA. LOURDES CABANSAY
G.R. No. 167345 November 23, 2007
Respondent Ma. Lourdes Cabansay (Cabansay) was hired as Senior Traning Manager of
ePacific Global Contact Center, Inc. Subsequently, she became a regular employee.
Respondent was tasked to prepare a new training process for the company’s Telesales
Trainees. After reviewing the training module prepared by respondent, Mr. Rosendo S.
Ballesteros (Ballesteros), the company’s Senior Vice President-Business Development
Group, found that the same did not contain any changes and that they were not ready to
present it. He thus instructed respondent through an electronic mail (e-mail) to
postpone the presentation and the implementation of the new training process.
Adversely reacting to respondent’s attitude because of the contents of reply through
email to him, (This is a very simple presentation and I WILL NOT POSTPONE it today,
its very easy to comprehend and as per YOUR INSTRUCTION we will be implementing
it next week, so when should we present this to the TLs? Lets not make SIMPLE
THINGS COMPLICATED. I will go on with the presentation this afternoon.), Ballesteros
sent Cabansay a memo on April 6, 2002, informing the latter that he found her message
to be a clear act of insubordination, causing him to lose his trust and confidence in her
as Manager of the Training Department. He then asked respondent to explain in writing
why she should not be terminated as a consequence of her acts.
Meanwhile, no presentation of the training module was made.
Clarifying that this was merely a case of miscommunication and that she had no
intention to disregard the order to postpone the implementation of the new training
process, Cabansay submitted two memoranda dated April 8 and 11, 2002.
However, on April 11, 2002, the same day she submitted her second explanation,
Cabansay received a memorandum from the HR Department/Office of the President
notifying her that she had been terminated from the service effective immediately for
having committed an act of insubordination resulting in the managements loss of trust
and confidence in her.
Respondent, thus, filed a case for illegal dismissal. She sought, among others, payment
of full backwages, separation pay, actual, moral and exemplary damages, cash
equivalent of vacation and sick leave, 13th month pay, and attorney’s fees.
LA: Dismissed the complaint.
NLRC: Affirmed LA’s decision.
CA: Cabansay is declared to have been illegally dismissed.
ISSUE:
Whether or not respondent Cabansay was illegally dismissed.
RULING:
No. The Decision of the Labor Arbiter, as affirmed by the NLRC, dismissing the
respondent’s complaint for illegal dismissal is REINSTATED.
In the case at bar, the reasonableness and lawfulness of Ballesteros’ order is not in
question, so is its relation to the duties of respondent. It should be noted that what is
involved in the directive is the new training process, which logically cannot be
implemented without being presented or communicated to the team leaders of the
company. Thus, when Ballesteros ordered the cessation of its implementation, there can
be no other inference than that he wanted to postpone the presentation of the training
process which was then already scheduled. Evident further in Ballesteros’ e-mail is that
he did not find any changes in the new module; hence, he wanted the implementation
thereof to be deferred and instructed respondent to consult with the other managers to
gather more input.
Be that as it may, respondent cannot belie the fact that she well-understood the directive
for her to postpone the presentation of the module, as she herself acknowledged in her
e-mail reply to SVP Ballesteros that she would discuss the new training process and
explain it to them in detail in the afternoon on that day, thus, she would not postpone
the scheduled presentation. There is no doubt, therefore, that the order of Ballesteros
was clearly made known to respondent.
As to the willfulness of her conduct, the same is manifest in her e-mail reply, which, as it
is written, is characterized by abject aggressiveness and antagonism: the e-mail has a
begrudging tone and is replete with capitalized words eliciting her resolve to indeed
contravene the SVPs directive.
Indeed, by refusing to postpone the presentation and implementation of the new
training process, respondent intentionally, knowingly and purposely, without justifiable
excuse, breached the trust and confidence reposed in her by her employer. To present
and discuss a training module, which is deemed by management as still inadequate in
its content, will certainly not only waste the time, effort and energy of the participants in
the discussion but will also entail losses on the part of the company.
It is of no moment that the presentation did not push through, and that no actual
damage was done by respondent to the company. The mere fact that respondent refused
to obey the reasonable and lawful order to defer the presentation and implementation of
the module already gave a just cause for petitioners to dismiss her.
Respondents conduct, in this case, is sufficient basis for the company to lose its trust
and confidence in her. Under the circumstances, the company cannot be expected to
retain its trust and confidence in and continue to employ a manager whose attitude is
perceived to be inimical to its interests. Unlike other just causes for dismissal, trust in
an employee, once lost, is difficult, if not impossible to regain.
 
ST. LUKE'S MEDICAL CENTER, INC. vs. ESTRELITO NOTARIO
G.R. No. 152166 : October 20, 2010
St. Luke's Medical Center, Inc. employed respondent as In-House Security Guard. His
work consisted mainly of monitoring the video cameras. Justin Tibon reported to the
management of petitioner hospital about the loss of his mint green traveling bag at the
time of respondent's duty. The cameras failed to record any incident of theft at room
257.
Respondent explained that on the subject dates, he was the only personnel on duty as
nobody wanted to assist him. Because of this, he decided to focus the cameras on the
Old and New Maternity Units, as these two units have high incidence of crime.
Finding the written explanation of respondent to be unsatisfactory, petitioner hospital
served on respondent a copy of the Notice of Termination.
Respondent filed a Complaint for illegal dismissal against petitioner hospital and its
Chairman. Labor Arbiter dismissed respondents complaint for illegal dismissal against
petitioners. On appeal by the respondent, the NLRC issued a Resolution reversing the
Decision of the Labor Arbiter. It stated that petitioners failed to submit proof that there
was an existing Standard Operating Procedure (SOP) in the CCTV monitoring system,
particularly on the focusing procedure.
Petitioners filed a Motion for Reconsideration, but the same was denied by the NLRC.
CA dismissed petitioners' petition for certiorari, affirming the NLRC's finding that while
respondent may appear to be negligent in monitoring the cameras on the subject dates,
the same would not constitute sufficient ground to terminate his employment.
HELD:
To effectuate a valid dismissal from employment by the employer, the Labor Code has
set twin requirements, namely: (1) the dismissal must be for any of the causes provided
in Article 282 of the Labor Code; and (2) the employee must be given an opportunity to
be heard and defend himself. This first requisite is referred to as the substantive aspect,
while the second is deemed as the procedural aspect.
An employer can terminate the services of an employee only for valid and just causes
which must be supported by clear and convincing evidence. The employer has the
burden of proving that the dismissal was indeed for a valid and just cause.
Under Article 282 (b) of the Labor Code, an employer may terminate an employee for
gross and habitual neglect of duties. Neglect of duty, to be a ground for dismissal, must
be both gross and habitual. Gross negligence connotes want of care in the performance
of ones duties. Habitual neglect implies repeated failure to perform one's duties for a
period of time, depending upon the circumstances. A single or isolated act of negligence
does not constitute a just cause for the dismissal of the employee. Under the prevailing
circumstances, respondent exercised his best judgment in monitoring the CCTV
cameras so as to ensure the security within the hospital premises. Verily, assuming
arguendo that respondent was negligent, although this Court finds otherwise, the lapse
or inaction could only be regarded as a single or isolated act of negligence that cannot be
categorized as habitual and, hence, not a just cause for his dismissal.
Where the dismissal was without just cause and there was no due process, Article 279 of
the Labor Code, as amended, mandates that the employee is entitled to reinstatement
without loss of seniority rights and other privileges and full backwages, inclusive of
allowances and other benefits, or their monetary equivalent computed from the time the
compensation was not paid up to the time of actual reinstatement.
The awards of separation pay and backwages are not mutually exclusive and both may
be given to respondent. An employee who is illegally dismissed is entitled to the twin
reliefs of full backwages and reinstatement. If reinstatement is not viable, separation
pay is awarded to the employee. In awarding separation pay to an illegally dismissed
employee, in lieu of reinstatement, the amount to be awarded shall be equivalent to one
month salary for every year of service.
LBC EXPRESS vs. MATEO
G.R. No. 168215 9 JUNE 2009

 James Mateo, designated as a customer associate, was a regular employee of LBC


Express (LBC), his job was to deliver and pick-up packages to and from LBC and
its customers
 Mateo used a Kawasaki motorcycle, one day, Mateo arrived at LBC’s Escolta
office, to drop off packages coming from various LBC air posts, he parked his
motorcycle directly in front of the LBC office, switched off the engine and took
the key with him
 However, he did not lock the steering wheel because he allegedly was primarily
concerned with the packages, including a huge sum of money that needed to be
immediately secured inside the LBC office, he returned promptly within three to
five minutes but the motorcycle was gone
 He immediately reported the loss to his superiors at LBC and to the nearest
police station, LBC, through its vice-president, Niño, directed Mateo to appear in
his office to explain his side and for formal investigation
 As directed, Mateo appeared and presented his side. After investigation, he
received a notice of termination from LBC. He was barred from reporting for
work. Mateofiled a complaint for illegal dismissal, payment of back wages and
reinstatement with damages
 Labor Arbiter and NLRC ruled in favor of LBC, but CA ruled that Mateo was
illegally dismissed
 LBC and Niño filed a petition to the Supreme Court seeking for the reversal of the
decision of the CA

ISSUE: Was Mateo illegally dismissed?


HELD: NO.

  He was terminated for being negligent which is one of the just and authorized
causes in the labor code
 The court ruled that he was grossly negligent in leaving the motorcycle without
locking it despite clear instruction not to do so
 The substantial amount of loss suffered by the company as a result of the
negligent act justifies dispensing the habituality requirement of the Labor Code
 The services of a regular employee may be terminated only for just or authorized
causes, including gross and habitual negligence under Article 282, paragraph (b)
of the Labor Code. Gross negligence is characterized by want of even slight care,
acting or omitting to act in a situation where there is a duty to act, not
inadvertently but willfully and intentionally with a conscious indifference to
consequences insofar as other persons may be affected.
 In this case, an employer cannot legally be compelled to continue with the
employment of a person admittedly guilty of gross negligence in the performance
of his duties. This holds true especially if the employee’s continued tenure is
patently inimical to the employer’s interest. What happened was not a simple
case of oversight and could not be attributed to a simple lapse of judgment.
 
MANSION PRINTING and CHENG vs. BITARA
G.R. No. 168120 January 25, 2012
Mansion Printing Center is a single proprietorship engaged in the printing of quality
self-adhesive labels, brochures, posters, stickers, packaging and the like.
Mansion engaged the services of Bitara as a helper (kargador). Respondent was later
promoted as the company’s sole driver tasked , among others, to deliver the products to
the clients within the delivery schedules.
Petitioners aver that the timely delivery of the products to the clients is one of the
foremost considerations material to the operation of the business. It being so, they
closely monitored the attendance of respondent. They noted his habitual tardiness and
absenteeism.
Petitioners issued a Memorandum requiring respondent to submit a written explanation
why no administrative sanction should be imposed on him for his habitual tardiness, to
which he replied. But despite respondent’s undertaking to report on time, however, he
continued to disregard attendance policies.
Respondent filed a complaint for illegal dismissal against the petitioners before the
Labor Arbiter.
ISSUE: Was there gross and habitual neglect of duty on the part of Bitara, warranting
his dismissal from service?
HELD: YES
There is no illegal dismissal. Valiao v. Court of Appeals is instructive:
xxx It bears stressing that petitioner’s absences and tardiness were not isolated
incidents but manifested a pattern of habituality. xxx The totality of infractions or
the number of violations committed during the period of employment shall be
considered in determining the penalty to be imposed upon an erring employee. The
offenses committed by him should not be taken singly and separately but in their
totality. Fitness for continued employment cannot be compartmentalized into tight little
cubicles of aspects of character, conduct, and ability separate and independent of each
other.
In Valiao, we defined gross negligence as “want of care in the performance of one’s
duties”and habitual neglect as “repeated failure to perform one’s duties for a period of
time, depending upon the circumstances.” These are not overly technical terms, which,
in the first place, are expressly sanctioned by the Labor Code of the Philippines, to wit:
ART. 282. Termination by employer. – An employer may terminate an employment for
any of the following causes: xxx (b) Gross and habitual neglect by the employee of his
duties; xxx
Bitara’s weekly time record for the first quarter of the year 2000 revealed that he came
late 19 times out of the 47 times he reported for work. He also incurred 19 absences out
of the 66 working days during the quarter. His absences without prior notice and
approval from March 11-16, 2000 were considered to be the most serious infraction of
all because of its adverse effect on business operations.
Clearly, even in the absence of a written company rule defining gross and habitual
neglect of duties, respondent’s omissions qualify as such warranting his dismissal from
the service.
NOTES:
We cannot simply tolerate injustice to employers if only to protect the welfare of
undeserving employees. As aptly put by then Associate Justice Leonardo A.
Quisumbing:
Needless to say, so irresponsible an employee like petitioner does not deserve a place in
the workplace, and it is within the management’s prerogative xxx to terminate his
employment. Even as the law is solicitous of the welfare of employees, it must also
protect the rights of an employer to exercise what are clearly management prerogatives.
As long as the company’s exercise of those rights and prerogative is in good faith to
advance its interest and not for the purpose of defeating or circumventing the rights of
employees under the laws or valid agreements, such exercise will be upheld.
CAVITE APPAREL, INCORPORATED vs.MICHELLE MARQUEZ
G.R. No. 172044, 06 February 2013
Michelle was hired as a regular employee in the Finishing Department.She committed 4
AWOL. When she reported back to work, she submitted the necessary medical
certificates. Nonetheless, Cavite Apparel suspended Michelle for 6 days. When Michelle
returned, Cavite Apparel terminated her employment for habitual absenteeism.
ISSUE: Whether or not a spread out four absences without leave constitutes gross and
habitual neglect and therefore a just cause for dismissal.
RULING:
No. Michelle’s four absences were not habitual; “totality of infractions” doctrine not
applicable. Neglect of duty, to be a ground for dismissal under Article 282 of the Labor
Code, must be both gross and habitual. Gross negligence implies want of care in the
performance of one’s duties. Habitual neglect imparts repeated failure to perform one’s
duties for a period of time, depending on the circumstances.
Four absences in her six years of service cannot be considered gross and habitual neglect
of duty, especially so since the absences were spread out over a six-month period.
Michelle’s penalty of dismissal too harsh or not proportionate to the infractions she
commited. Although Michelle was fully aware of the company rules regarding leaves of
absence, and her dismissal might have been in accordance with the rules, it is well to
stress that the Supreme Court is not bound by such rules.
While management’s prerogative is recognized to discipline its employees, the exercise
of this prerogative should at all times be reasonable and should be tempered with
compassion and understanding. Dismissal is the ultimate penalty that can be imposed
on an employee. Where a penalty less punitive may suffice, whatever missteps may be
committed by labor ought not to be visited with a consequence so severe for what is at
stake is not merely the employee’s position but his very livelihood and perhaps the life
and subsistence of his family.
ESSENCIA MANARPIIS vs. TEXAN PHILIPPINES, INC.
January 28, 2015           GR. NO. 197011
 
Texan Philippines, Inc. (TPI) is engaged in the importation, distribution and marketing
of imported fragrances and aroma and other specialized products and services. It hired
Essencia Q. Manarpiis (petitioner) as Sales and Marketing Manager. She was later
dismissed on July 25, 2000 when she received a notice of termination on the ground of
dishonesty, loss of confidence based on alleged collusion in defrauding the company
financed. Another ground was also for abandonment of work.
Claiming insurmountable losses, respondents served a written notice addressed to all
their employees that TPI will cease operations by August 31, 2000.
 
Petitioner filed a complaint for illegal dismissal, non-payment of overtime pay, holiday
pay, service incentive leave pay, unexpired vacation leave and 13th month pay and with
prayer for moral and actual damages.
 
Respondent asserted that the requisite notices of business closure to government
authorities and to their employees were complied with, and notwithstanding that TPI
has in fact continued its operations, petitioner was found to have committed infractions
resulting in loss of confidence which was the ground for the termination of her
employment.
 
ISSUE:
Whether Petitioner was illegally dismissed
 
HELD:
Yes. Petitioner was dismissed without just or authorized cause, and that the announced
cessation of business operations was a mere subterfuge for getting rid of petitioner. The
CA’s finding of serious business losses is not borne by the evidence on record. The
financial statements supposedly bearing the stamp mark of BIR were not signed by an
independent auditor. Besides, the non-compliance with the requirements under Article
283 of the Labor Code, as amended, gains relevance in this case not for the purpose of
proving the illegality of the company closure or cessation of business, which did not
materialize, but as an indication of bad faith on the part of respondents in hastily
terminating petitioner’s employment.
Under the circumstances, the subsequent investigation and termination of petitioner on
grounds of dishonesty, loss of confidence and abandonment of work, clearly appears as
an afterthought as it was done only after petitioner had filed an illegal dismissal case
and respondents have been summoned for hearing before the LA.
The court laid down the two elements which must concur for a valid abandonment:
(1) the failure to report to work or absence without valid or justifiable reason, and
(2) a clear intention to sever the employer-employee relationship, with the second
element as the more determinative factor being manifested by some overt acts.
 
Abandonment as a just ground for dismissal requires the deliberate, unjustified refusal
of the employee to perform his employment responsibilities. Mere absence or failure to
work, even after notice to return, is not tantamount to abandonment. Furthermore, it is
well-settled that the filing by an employee of a complaint for illegal dismissal
with a prayer for reinstatement is proof enough of his desire to return to
work, thus, negating the employer’s charge of abandonment. An employee
who takes steps to protest his dismissal cannot logically be said to have abandoned his
work.
 
On the issue of loss of confidence, an employer has its own interest to protect, and
pursuant thereto, it may terminate a managerial employee for a just cause, such
prerogative to dismiss or lay off an employee must be exercised without abuse of
discretion. Indeed, the consistent rule is that if doubts exist between the evidence
presented by the employer and the employee, the scales of justice must be tilted in favor
of the latter. The employer must affirmatively show rationally adequate evidence that
the dismissal was for justifiable cause. Thus, when the breach of trust or loss of
confidence alleged is not borne by clearly established facts, as in this case,
such dismissal on the cited grounds cannot be allowed.
 
PHILIPPINE AIRLINES vs. NLRC
G.R. No. 126805 March 16, 2000
 
STA. ANA vs. MANILA JOCKEY CLUB, INC.
G.R. No. 208459 February 15, 2017
 
In May 1977, MJCI, a corporation with legislative franchise to operate horse race
betting, hired Julieta B. Sta. Ana as outlet teller of its off-track betting station in
Tayuman, Manila. On November 13, 2008, however, MTCI issued a Memorandum
stating that its Treasury Department was discovered to have been illegally appropriating
funds and lending it out to the employees of MJCI. As a result, MTCI required its
officers and employees to report any loan obtained from said department or any of its
personnel. On December 21, 2008, MJCI’s Internal Auditing Department submitted its
Preliminary Report indicating that its Agudo OTB Branch had unaccounted check
remittances amounting to ₱44,377,455.00 for the period January 10, 2008 to November
30, 2008.
MJCI charged Sta. Ana with the following infractions:
(a) Stealing or attempting to steal corporate property or money/corporate assets;
(b) Malversation;
(c) Engaging/conniving in anomalous transactions. Later, MJCI served upon Sta. Ana a
Notice of Investigation reiterating the accusations against her, and narrating the
circumstances surrounding her case that there were unaccounted shortages incurred by
the Cashier Department. The Balance Sheet as of November 2008 indicated that the
Cash on Hand amounted to around ₱198 million; actual counting of the cash in vault
revealed, however, that the actual amount is only around ₱3.1 million. At the center of
this irregularity and/or fraud is Josephine Tejada. That Josephine Tejada, without
authority, has been lending large amount of money to some MJCI personnel using
corporate funds. It has likewise been reported that Sta. Ana were abetting Josephine
Tejada in the said unauthorized lending or that you yourself has also been lending to
some MJCI personnel using corporate funds and without any authority from
management. The Notice further informed Sta. Ana of her 30-day suspension without
pay effective January 16, 2009.
Consequently, the SDC found Sta. Ana guilty of conspiring to defraud, illegally take
funds, and cause irreparable damage to MJCI; as such, MJCI lost its trust on her. It also
declared that even granting that there was no conspiracy, Sta. Ana, nonetheless,
committed gross inexcusable negligence for failure to perform her duties and protect the
interest of MJCI. SDC recommended the dismissal of Sta. Ana and the filing of criminal
cases for qualified theft and other appropriate charges. On February 16, 2009, MJCI
issued a Notice of Termination to Sta. Ana In her Answer, Sta. Ana averred that she did
not know anything regarding MJCI's unaccounted money and that her suspension was
unjust. She maintained that she did not violate any company rule by engaging in the
lending business. Sta. Ana asserted that she had been in the money lending business for
15 years, or even prior to the takeover by the new management of MJCI, and that her
capital was sourced from the sale of her fishing boats. On February 25, 2009, Sta. Ana
filed a Complaint for illegal dismissal and payment of actual, moral and exemplary
damages, and attorney's fees against MJCI/ Atty. Reyno, its President.
Sta. Ana averred that she had been in the service for 31 years prior to her dismissal. She
stressed that she had bank deposits, real properties and fishing business to fund her
lending business; and, the fact that she lent money to her co-employees is not proof that
she used MJCI's funds for her business. She further insisted that there was no company
rule prohibiting employees from engaging in their own businesses. In addition, Sta. Ana
contended that she had no direct access to her employer's money; thus, she could not
have stolen it. She pointed out that she never incurred a shortage in remitting the
income of her OTB Branch or the OTB Tayuman Branch. Lastly, Sta. Ana stated that her
one-time request for Tejada to accommodate a company personnel of the company is
not evidence of any complicity with Tejada. Similarly, she should not be dragged into the
controversy in the Cashier/Treasury Department of MJCI just because she was a
"kumare" of Tejada.
Sta. Ana submitted additional documents to show her capability to engage in loan
operations: These are:(1) Certification from PS Bank that Sta. Ana has existing housing
loan with outstanding balance of ₱439,421.65, (2) Permit to Operate fishing vessels
issued by the Maritime Industry Authority, (3) various statement of accounts from BPI,
HSBC, Citibank, BDO, Standard Chartered, Metrobank credit cards. The three fishing
vessels were already sold, according to her, and she used the proceeds in her lending
business. Likewise, the annotations in Transfer Certificate of Title No. T-389599 under
the name of Sta. Ana and her spouse proved that they had been mortgaging their real
property since 2003. The latest of such mortgage was on August 23, 2007 to secure the
loan of One Million Pesos from PSBank.
ISSUE: 
Whether or not Sta. Ana was validly dismissed on the ground of loss of trust and
confidence. 
HELD:
No, Sta. Ana was not validly dismissed on the ground of loss of trust and confidence.
The court held that it is a cardinal rule that loss of trust and confidence should be
genuine, and not simulated; it must arise from dishonest or deceitful conduct, and must
not be arbitrarily asserted in the face of overwhelming contrary evidence.44While proof
beyond reasonable doubt is not required, loss of trust must have some basis or such
reasonable ground for one to believe that the employee committed the infraction, and
the latter's participation makes him or her totally unworthy of the trust demanded by
the position. Here, MJCI failed to prove that Sta. Ana committed willful breach of its
trust.1 Particularly, it failed to establish that Sta. Ana used its employee for her personal
business during office hours, and used its money; without authority, to lend money to
another. Hence, to dismiss her on the ground of loss of trust and confidence is
unwarranted.
REXIE A. HORMILLOSA, Petitioner, v. COCA-COLA BOTTLERS
PHILS., INC.
G.R. No. 198699 : October 9, 2013
Petitioner Hormillosa was employed as a route salesman by Coca-Cola Bottlers Phils.,
Inc. (CBPI). His duties included, among others, selling CBPIs soft drink products, either
on cash or on credit basis; receiving payments from proceeds of the sale or payments of
past due or current accounts; issuing sales invoices; and receiving empty bottles and
cases of soft drinks (empties).
Concerning the sales invoices, he was authorized to issue them on a cash and credit
basis. He prepared the invoices stating the names of the customers, the quantity and
kind of merchandise purchased, and the corresponding amounts. He was required to
make the customers sign the invoices, especially in cases they were on credit basis, and
leave copies with them. The invoices were then submitted to the Finance Department for
accounting and auditing.
The then CBPI District Sales Supervisor, Raul S. Tiosayco III conducted a verification
and audit of the accounts handled by Hormillosa. He discovered transactions in
violation of CCBPI Employee Code of Disciplinary Rules and Regulations, specifically
Fictitious sales transactions and fictitious issuances; Falsification of company
records/data/documents/invoices/reports; non-issuance or mis-issuance of invoices
and receipts as well as commercial documents to dealers; forgery; misuse, abuse or
defalcation of funds form market development program.
After due notice and hearing, petitioner was terminated in his employment. Hormillosa
filed a complaint for illegal dismissal.
The LA dismissed Hormillosas complaint ruling that his termination was proper
however, awarded him a separation pay, citing the case of Magos v. NLRC, 360 Phil. 670
(1998),where it was stated that separation pay could be granted as a form of equitable
relief even if the dismissal was for a just cause.
On appeal, the NLRC ordered the remand of the case to the SRAB to give Hormillosa the
opportunity to confront the witnesses and evidence against him. Moreover, it stated that
Section 5(b), Rule V of the 1990 NLRC Rules was not observed.
The SRAB, this time through LA Acosta, ruled that Hormillosa was illegally dismissed
but did not order his reinstatement due to strained relations. It was decreed that he was
entitled to backwages and separation pay.
CBPI appealed to the NLRC, arguing that the decision of LA Acosta was bereft of factual
findings, applicable laws and legal principles. It insisted that the dismissal of Hormillosa
was proper considering that the charges against him were proven by substantial
evidence.
The NLRC upheld the decision of LA Acosta. CBPI moved for reconsideration but its
motion was denied.
CBPI elevated the matter to the CA via a petition for certiorari under Rule 65.
The CA nullified and set aside the NLRC decision and held that the dismissal of
Hormillosa was valid. According to the CA, Hormillosa was validly dismissed under
Article 282 (c) of Labor Code, as amended. It states that loss of confidence applies to
cases involving employees who occupy positions of trust and confidence or to those
situations where the employee is routinely charged with the care and custody of the
employers money or property.The CA pointed out that there were established
circumstances proving such breach of trust and confidence.
Hence, the latter is validly dismissed from his employment. Without finding of illegal
dismissal, the monetary awards bestowed on him by the SRAB No. VI and modified by
public respondent have no basis.
ISSUE : Whether or not the Court of Appeals erred in validly dismissing the Petitioner.
HELD:
The rule is that, in labor cases, substantial evidence or such relevant evidence as a
reasonable mind might accept as sufficient to support a conclusion is required.The CA
was correct when it ruled that Hormillosas employment was validly terminated under
paragraph (c) of Art. 282 of the Labor Code (i.e., Fraud or willful breach by the
employee of the trust reposed in him by his employer or duly authorized representative).
There was substantial evidence to justify his dismissal.
In Bristol Myers Squibb (Phils.), Inc. v. Baban, G.R. No. 167449, December 17, 2008the
Court discussed the requisites for a valid dismissal on the ground of loss of trust and
confidence as follows.
It is clear that Article 282(c) of the Labor Code allows an employer to terminate the
services of an employee for loss of trust and confidence. The right of employers to
dismiss employees by reason of loss of trust and confidence is well established in
jurisprudence.
The first requisite for dismissal on the ground of loss of trust and confidence is that the
employee concerned must be one holding a position of trust and confidence.
There are two (2) classes of positions of trust. The first class consists of managerial
employees. They are defined as those vested with the powers or prerogatives to lay down
management policies and to hire, transfer suspend, lay-off, recall, discharge, assign or
discipline employees or effectively recommend such managerial actions. The second
class consists of cashiers, auditors, property custodians, etc. They are defined as those
who in the normal and routine exercise of their functions, regularly handle significant
amounts of money or property.
The second requisite is that there must be an act that would justify the loss of trust and
confidence. Loss of trust and confidence to be a valid cause for dismissal must be based
on a willful breach of trust and founded on clearly established facts. The basis for the
dismissal must be clearly and convincingly established but proof beyond reasonable
doubt is not necessary. Garcia v. National Labor Relations Commission, G.R. No.
113774, April 15, 1998.
Hormillosa, being a route salesman, falls under the second class. By selling soft drink
products and collecting payments for the same, he was considered an employee who
regularly handled significant amounts of money and property in the normal and routine
exercise of his functions.
Clearly, Hormillosa occupies a position of trust. As correctly pointed out by the CA,
there was a high degree of trust and confidence reposed on him and when this
confidence was breached, the employer was justified in taking the appropriate
disciplinary action.
With regard to the second requisite for dismissal on the ground of loss of trust and
confidence, the Court finds that Hormillosa committed acts which warranted his
dismissal from employment.
The award of separation pay is authorized in the situations dealt with in Article 283 and
Art. 284 of the Labor Code, but not in terminations of employment based on instances
enumerated in Art. 282.
"The only cases when separation pay shall be paid, although the employee was lawfully
dismissed, are when the cause of termination was not attributable to the employee's
fault but due to : (1) the installation of labor saving devices, (2) redundancy, (3)
retrenchment, (4) cessation of employer's business, or (5) when the employee is
suffering from a disease and his continued employment is prohibited by law or is
prejudicial to his health and to the health of his co-employees (Articles 283 and 284,
Labor Code.) Other than these cases, an employee who is dismissed for a just and lawful
cause is not entitled to separation pay even if the award were to be called by another
name." Eastern Paper Mills, Inc. v. NLRC, 252 Phil. 618, 621 (1989).
In the case, the cause for the dismissal from employment of Hormillosa clearly falls
under Article 282 of the Labor Code. Therefore, he is not entitled to any separation pay.
Cecilia Manese v. Jollibee Foods Corp.
G.R. No: 170454 October 11, 2012
1. Petitions were a team of employees of Jollibee who were tasked to open a new Jollibee
branch at Festival Mall on December 12, 2000.
2. In preparation for the opening of the new branch, petitioner Julietes Cruz requested
the Commissary Warehouse and Distribution (commissary) for the delivery of wet and
frozen goods on December 9, 2000 to comply with the 30-day thawing process of the
wet goods, particularly "Chickenjoy."
3. The opening of the store was postponed thrice. When the opening was rescheduled to
December 24, 2000, petitioner Cruz made another requisition for the delivery of the
food on December 23, 2000, but the opening date was again postponed.
4. The engineering team assured the operations manager (respondent Elizabeth Cruz)
that the store would open on December 28, 2000. Upon advice of the Opening Team
Manager, Julietes did not cancel the request for delivery of products.
5. The following events took place:

a. December 23 – 450 packs (4,500 peices) of chickenjoy were delivered and placed
in the freezer.
b. December 26 – Julietes thawed the 450 packs (4,500 peices) of chickenjoy for the
branch opening on the 28th.

6. Shelf life of chickenjoy is 25 days from being marinated, and should be served on the
3rd day upon thawing (so the 29th). The shelf life of the Chickenjoy is 25 days from the
time it is marinated; and, once thawed, it should be served on the third day. Its shelf life
cannot go beyond three days from thawing. After that, the remaining Chickenjoy
products are no longer served, and they are packed in plastic and placed in a garbage
bag to be stored in the freezer. Within the period provided for in the company policy,
valid Chickenjoy rejects are usually returned to the commissary, while rejects which are
unreturnable are wasted and disposed of properly.
7. Despite postponements of the store's opening, the store's sales targets for December
28 and 29, 2000, considered peak times, were not revised by the operations manager.
The sales targets of P200,000.00 for the first day and P225,000.00 for the second day
were not reached, as the store's actual sales were only P164,000.00 and P159,000.00,
respectively.
8. Sometime in January 2001, Julietes attempted to return 150 pieces of Chickenjoy
rejects to Commissary but the driver refused to accept them due to discoloration,
deteriorated condition, and fear that it may be charged against him, which led to the
chickens being returned to the freezers. 
9. During the first week of March 2001, their team had a meeting on what to do with the
chicken – soak and clean the rejects in soda water and segregate the valid rejects from
the wastes.
10. On April 2, 2001, petitioner Cruz was transferred to Jollibee Shell South Luzon
Tollway branch. She estimated that the total undisposed Chickenjoy rejects from the
450 packs delivered on December 23, 2000 was only about 1,140 pieces as of January
2001. She failed to make the proper indorsement as the area manager directed her to
report immediately to her new assignment.
11. The area manager, Divina Evangelista, visited the Festival branch and saw Eufemio
Penano cleaning the chickens and told the latter to dispose it but the former replied that
they be allowed to return them to the Commissary. A corporate audit was conducted and
found that 2,130 pieces of the rejects were declared wastage.
12. Divina then issued a memo requiring petitioners to explain why they should not be
meted the appropriate penalty for violation of Code of Discipline for extremely serious
misconduct; gross negligence; product tampering; and fraud or falsification of company
records and insubordination in connection with their findings that 2,130 pieces of
Chickenjoy rejects were kept inside the walk-in freezer, which could cause product
contamination and threat to food safety.
13. Explanations of petitioners:
Cecilia Manese
Foul smell and discoloration of the Chickenjoy rejects were due to the breakdown of the
walk-in facilities prior to the store’s grand opening. During that time, the store was
using temporary power supply, so that it could open during Christmas Day and the
Metro Manila Film Festival.
She admitted that she was not able to immediately inform Area Manager about it.
They did their best, but they were not able to save a bulk of the said Chickenjoy due to
the holiday season.
They asked for assistance from other stores, but they could only accommodate a few
stocks, as most of their storage areas were filled with their own stocks.
They did not immediately dispose of the Chickenjoy rejects out of fear of being
reprimanded and it would add to the existing problems of the branch regarding low
sales and profit.
Eufemio Penamo
He was not familiar with managerial duties in the kitchen since he had no proper
training, hence, he only followed Cecilia Manase’s instructions.
Julietes Cruz
Before her transfer, there were only 1,200 rejects
Some were greenish because they were the ones delivered when the walk-in freezers
were still on pre-setting temperature and operating on temporary power.
14. The management conducted an investigation and sent petitioners notifying them
that they are terminated due to loss of trust and confidence.
15. Petitioners filed a complaint for illegal dismissal.
16. LA’s Decision:

a. Cecilia Manese & Eufemio Penano: Dismissed


b. Julietes Cruz: Illegal Dismissal (He was no longer working and was transferred.)
17. NLRC’s Decision: Affirmed LA

a. Julietes: Actually, it was a valid dismissal but management failed to file a timely
appeal.

18. CA’s Decision: Valid dismissal


ISSUE #1: WoN CA acted with grave abuse of discretion in passing upon the
legality of Cruz’ dismissal considering the LA decision has become final and
executory since there was no timely appeal filed by Jollibee.
HELD -  YES, CA acted with grave abuse. Failure to appeal from the decision of the
Labor Arbiter renders the decision on the illegal dismissal of Cruz final and executory.
1. Jollibee did not appeal from the decision of the Labor Arbiter who ruled that the
dismissal of petitioner Cruz was illegal. They only filed an Opposition to Appeal, which
prayed for the reversal of the Labor Arbiter’s orders declaring as illegal the dismissal of
Cruz and directing payment of her separation pay.

a. The LA’s decision was received on August 28 and they had 10 days (September 8)
to file an appeal.
b. Instead of an appeal, they filed this opposition which could have been treated as
an appeal but was filed beyond the 10 days, around October.

ISSUE #2: WoN CA misappreciated the facts when it affirmed the dismissal
on the ground of loss of trust and confidence (managerial employees) – No,
CA did not misappreciate the facts.
Petitioner’s Argument:
The area manager conducted a store audit and had favorable finding – all departments,
including food stock and food safety, was fair and satisfactory negated the charge of loss
of trust and confidence.
HELD

1. The respective memorandum with a notice of termination given to each of the


petitioners clearly expressed that their respective acts and omissions enumerated
made respondent company lose its trust and confidence in petitioners, who were
managerial employees; hence, they were terminated from employment.
2. The mere existence of a basis for the loss of trust and confidence justifies the
dismissal of the managerial employee because when an employee accepts a
promotion to a managerial position or to an office requiring full trust and
confidence, such employee gives up some of the rigid guaranties available to
ordinary workers.
3. Proof beyond reasonable doubt is not required provided there is a valid reason
for the loss of trust and confidence, such as when the employer has a reasonable
ground to believe that the managerial employee concerned is responsible for the
purported misconduct and the nature of his participation renders him unworthy
of the trust and confidence demanded by his position
a. Right of the management to dismiss must be balanced against the managerial
employee’s right to security of tenure which is not one of the guaranties he gives
up.
b. Managerial employees enjoy security of tenure and, although the standards for
their dismissal are less stringent, the loss of trust and confidence must be
substantial and founded on clearly established facts sufficient to warrant the
managerial employee’s separation from the company.

4. In this case, the acts listed in the memo were valid bases.
ISSUE #3: WoN Manese’s unpaid balance on her car loan can be offset with
the monetary claims due to her? – NO.

1. In Nestlé Philippines, Inc. v. NLRC –the employer's demand for payment of the
employees' amortization on their car loans, or, in the alternative, the return of the
cars to the employer, is not a labor, but a civil dispute. It involves debtor-creditor
relations, rather than employee-employer relations.
2. Manese has an obligation to pay the balance on the car loan to Jollibee. If she
cannot afford to pay the balance, she can return the car to Jollibee. Otherwise,
Jollibee can file a civil case for the payment of the balance on the car loan or for
the return of the car. The legal remedy of the respondent company is civil in
nature, arising from a contractual obligation.

 
RENO FOODS, INC. v. NAGKAKAISANG LAKAS NG MANGGAGAWA
GR 164016 / MARCH 15, 2010 

•         Reno Foods is a manufacturer of canned meat products. Vicente Khu is the


president. NenitaCapor was an employee of Reno.
•         A guard on duty found six Reno canned goods wrapped in nylon leggings inside
Capor’s fabric clutch bag. (The other contents of the bag were money and a small
medicine container.)
•         Reno gave Capor several opportunities to explain her side. After they sent her a
Notice of Termination, she was given an opportunity for reconsideration through a
labor-management grievance conference. However, Reno did not find reason to
change its decision to terminate her employment.
•         Afterwards, Reno filed a complaint against Capor for qualified theft. A Resolution
was issued finding probable cause for the crime charged.
•         The NLM-Katipunan filed on behalf of Capor a complaint for illegal dismissal, and
prayed that she be paid her full backwages as well as moral and exemplary
damages.
•         LA: guilty of serious misconduct (cause for termination); not entitled to separation
pay. NLRC affirmed, added award for financial assistance. CA affirmed NLRC, and
stressed the primacy of laborer’s welfare.
•         On the date that the CA issued its Decision, Capor filed a Manifestation that she
was acquitted of the charge against her (due to lack of substantial evidence on
Reno’s part).

 ISSUES & RATIO. WON financial assistance should be granted to an employee


validly dismissed for theft of company property –NO :(

 
•         Capor’s acquittal in the criminal case –one grounded on the existence
of reasonable doubt –will not preclude the determination in a labor
case that she is guilty of the acts inimical to Reno’s interests.
•         Criminal cases require proof beyond reasonable doubt while labor
disputes require only substantial evidence (relevant evidence as a
reasonable mind might accept as adequate to justify a conclusion).
•         The lower courts found substantial evidence to conclude that Capor had been
validly dismissed for dishonesty or serious misconduct.
•         RE SEPARATION PAY:Separation pay is only warranted when the cause for
termination is not attributable to the employee’s fault (Art. 283 and 284), as well as
illegal dismissal in which reinstatement is no longer feasible.
•         Jurisprudence has classified theft of company property as a serious misconduct.
•         It is true that there have been instances when the Court awarded financial
assistance to employees who were terminated for just causes, on grounds of equity
and social justice. But in those cases, the Court recognized “the harsh realities faced
by employees that forced them to violate company policies.”
•         But when the employee commits an act of dishonesty, depravity or iniquity, the
grant of financial assistance is misplaced compassion. It is tantamount not only to
condoning a patently illegal or dishonest act, but an endorsement thereof. It will be
an insult to all the laborers who, despite their economic difficulties, strive to
maintain good values and moral conduct.
•         Length of service (Capor served for 39 years) and a previously clean employment
record cannot simply erase the gravity of the betrayal exhibited by a malfeasant
employee.

LYNVIL FISHING ENTERPRISES, INC. vs. ANDRES G. ARIOLA


G.R. No. 181974 : February 1, 2012
Lynvil Fishing Enterprises, Inc. (Lynvil) is a company engaged in deep-sea fishing,
operating along the shores of Palawan and other outlying islands of the Philippines.
Lynvil received a report from Romanito Clarido, one of its employees, that on 31 July
1998, he witnessed that while on board the company vessel Analyn VIII, Lynvil
employees conspired with one another and stole eight (8) tubs of "pampano" and
"tangigue" fish and delivered them to another vessel, to the prejudice of Lynvil.
By reason of the report and after initial investigation, Lynvils General Manager Rosendo
S. De Borja (De Borja) summoned respondents to explain within five (5) days why they
should not be dismissed from service. Failing to explain as required, respondents
employment was terminated.
Lynvil, through De Borja, filed a criminal complaint against the dismissed employees for
violation of P.D. 532, or the Anti-Piracy and Anti-Highway Robbery Law of 1974 before
the Office of the City Prosecutor of Malabon City. First Assistant City Prosecutor
Rosauro Silverio found probable cause for the indictment of the dismissed employees
for the crime of qualified theft.
Upon being informed about this, Ariola, Calinao, Nubla and Sebullen went to the Lynvil
office. However, they were told that their employments were already terminated.
Aggrieved, the employees filed with the Arbitration Branch of the National Labor
Relations Commission-National Capital Region on 25 August 1998 a complaint for
illegal dismissal with claims for backwages, salary differential reinstatement, service
incentive leave, holiday pay and its premium and 13th month pay from 1996 to1998.
They also claimed for moral, exemplary damages and attorneys fees for their dismissal
with bad faith.
Labor Arbiter Ramon Valentin C. Reyes found merit in complainants charge of illegal
dismissal. NLRC reversed and set aside the Decision of the Labor Arbiter. The private
respondents except Elorde Baz filed a Petition for Certiorari before the Court of Appeals
alleging grave abuse of discretion on the part of NLRC. The Court of Appeals found
merit in the petition and reinstated the Decision of the Labor Arbiter except as to the
award of attorneys fees. Hence, this petition.
ISSUE: Whether or not the dismissal was illegal
HELD: No.
An employer may terminate an employment based on fraud or willful breach of the trust
reposed on the employee.
The Labor Code provides that an employer may terminate an employment based on
fraud or willful breach of the trust reposed on the employee. Such breach is considered
willful if it is done intentionally, knowingly, and purposely, without justifiable excuse, as
distinguished from an act done carelessly, thoughtlessly, heedlessly or inadvertently. It
must also be based on substantial evidence and not on the employers whims or caprices
or suspicions otherwise, the employee would eternally remain at the mercy of the
employer. Loss of confidence must not be indiscriminately used as a shield by the
employer against a claim that the dismissal of an employee was arbitrary. And, in order
to constitute a just cause for dismissal, the act complained of must be work-related and
shows that the employee concerned is unfit to continue working for the employer.
In addition, loss of confidence as a just cause for termination of employment is
premised on the fact that the employee concerned holds a position of responsibility,
trust and confidence or that the employee concerned is entrusted with confidence with
respect to delicate matters, such as the handling or care and protection of the property
and assets of the employer. The betrayal of this trust is the essence of the offense for
which an employee is penalized.
Breach of trust is present in this case.
We cannot close our eyes to the positive and clear narration of facts of the three
witnesses to the commission of qualified theft. Jonathan Distajo, a crew member of the
Analyn VIII, stated in his letter addressed to De Borja dated 8 August 1998, that while
the vessel was traversing San Nicolas, Cavite, he saw a small boat approach them. When
the boat was next to their vessel, Alcovendas went inside the stockroom while Sebullen
pushed an estimated four tubs of fish away from it. Ariola, on the other hand, served as
the lookout and negotiator of the transaction.
Finally, Baz and Calinao helped in putting the tubs in the small boat. He further added
that he received P800.00 as his share for the transaction. Romanito Clarido, who was
also on board the vessel, corroborated the narration of Distajo on all accounts in his 25
August 1998 affidavit. He added that Alcovendas told him to keep silent about what
happened on that day. Sealing tight the credibility of the narration of theft is the
affidavit executed by Elorde Baz dated 3 May 1999. Baz was one of the dismissed
employees who actively participated in the taking of the tubs. He clarified in the affidavit
that the four tubs taken out of the stockroom in fact contained fish taken from the eight
tubs. He further stated that Ariola told everyone in the vessel not to say anything and
instead file a labor case against the management. Clearly, we cannot fault Lynvil and De
Borja when it dismissed the employees.
JOHN HANCOCK LIFE INSURANCE vs. JOANNA CANTRE DAVIS
G.R. No. 169549, September 3, 2009
Respondent Joanna Cantre Davis was agency administration officer of petitioner John
Hancock Life Insurance Corporation.
On October 18, 2000, Patricia Yuseco, petitioner’s corporate affairs manager, discovered
that her wallet was missing. She immediately reported the loss of her credit cards to AIG
and BPI Express. To her surprise, she was informed that “Patricia Yuseco” had just
made substantial purchases using her credit cards in various stores in the City of
Manila. She was also told that a proposed transaction in Abenson’s-Robinsons Place was
disapproved because “she” gave the wrong information upon verification.
Because loss of personal property among its employees had become rampant in its
office, petitioner sought the assistance of the National Bureau of Investigation (NBI).
The NBI, in the course of its investigation, obtained a security video from Abenson’s
showing the person who used Yuseco’s credit cards. Yuseco and other witnesses
positively identified the person in the video as respondent.
Consequently, the NBI and Yuseco filed a complaint for qualified theft against
respondent in the office of the Manila city prosecutor. But because the affidavits
presented by the NBI (identifying respondent as the culprit) were not properly verified,
the city prosecutor dismissed the complaint due to insufficiency of evidence.
Meanwhile, petitioner placed respondent under preventive suspension and instructed
her to cooperate with its ongoing investigation. Instead of doing so, however,
respondent filed a complaint for illegal dismissal alleging that petitioner terminated her
employment without cause.
ISSUE: Whether or not petitioner validly dismissed respondent for cause analogous to
serious misconduct.
RULING: Article 282(e) of the Labor Code talks of other analogous causes or those
which are susceptible of comparison to another in general or in specific detail. For an
employee to be validly dismissed for a cause analogous to those enumerated in Article
282, the cause must involve a voluntary and/or willful act or omission of the employee.
A cause analogous to serious misconduct is a voluntary and/or willful act or
omission attesting to an employee’s moral depravity. Theft committed by an
employee against a person other than his employer, if proven by substantial
evidence, is a cause analogous to serious misconduct.
The labor arbiter and the NLRC relied not only on the affidavits of the NBI’s witnesses
but also on that of respondent. They likewise considered petitioner’s own investigative
findings.
 
MODULE 10 – AUTHORIZED CAUSES, RELIEFS IN ILLEGAL
DISMISSAL & RETIREMENT
ARABIT v JARDINE PACIFIC

Petitioners were former regular employees of respondent Jardine. Petitioner were officers and
members of a legitimate labor union and the SEBA of the employees.

Jardine decided to reorganize and implement a redundancy program among its employees on
the ground of alleged financial losses. Among those affected by said program were the
petitioners. Thereafter, contractual employees to perform the functions of said employees were
hired by Jardine.

Later, an amicable settlement was reached between the union and Jardine wherein petitioners
accepted their redundancy pay without prejudice to their right to question the legality of their
dismissal with the NLRC. A separation package was then paid to the petitioners.

PETITIONERS WERE ILLEGALLY DISMISSED. Retrenchment and redundancy are two


different concepts; thus, they should not be used interchangeably. 

Redundancy exists where the services of an employee are in excess of what is reasonably
demanded by the actual requirements of the enterprise. A position is redundant where it is
superfluous, and superfluity of a position or positions may be the outcome of a number of
factors. Retrenchment, on the other hand, is used interchangeably with the term "lay-off." It is an
act of the employer of dismissing employees because of losses in the operation of a business,
lack of work, and considerable reduction on the volume of his business, a right consistently
recognized and affirmed by this Court. 

For the implementation of a redundancy program to be valid, the employer must comply with the
following requisites: (1) written notice served on both the employees and the Department of
Labor and Employment at least one month prior to the intended date of retrenchment; (2)
payment of separation pay equivalent to at least one month pay or at least one month pay for
every year of service, whichever is higher; (3) good faith in abolishing the redundant positions;
and (4) fair and reasonable criteria in ascertaining what positions are to be declared redundant
and accordingly abolished. 

At two levels (requisites 3 and 4), Jardine failed to set the required fair and reasonable criteria in
the termination of the petitioners’ employment, leading to the conclusion that the termination
from the service was arbitrary and in bad faith. 

SHIMIZU PHILIPPINES v CALLANTA


Respondent was employed by respondent for its Yutaka-Giken Project. In a memo, respondent
was later informed that his services will be terminated on the ground of lack of any vacancy in
other projects and the need to re-align the company’s personnel requirements due to maximum
financial commitments. An illegal dismissal complaint was then filed by the respondent.

Petitioner contends that the termination was in accordance with a valid retrenchment program
being implemented since 1996 due to financial crisis. On the other hand, respondent claims that
petitioner failed to comply with the requirements for retrenchment and that it failed to use fair
and reasonable criteria in determining which employees shall be retrenched or retained. 
 
RETRENCHMENT PROGRAM WAS IMPLEMENTED IN GOOD FAITH. Petitioner undertook
several measures in cutting down its costs. Petitioner was able to prove that it incurred
substantial business losses, that it offered to pay respondent his separation pay, that the
retrenchment scheme was arrived at in good faith, and lastly, that the criteria or standard used
in selecting the employees to be retrenched was work efficiency which passed the test of
fairness and reasonableness. 

ASIAN ALCOHOL CORP. v NLRC

The Parson family sold their majority rights over Asian Alcohol to Prior Holdings, Inc. due to
mounting business losses. To thwart further losses, Prior Holdings implemented an
organizational plan and other cost-saving measures. Some one 117 employees out of a total
workforce of 360 were separated. 72 of them occupied redundant positions that were
abolished. 

The private respondents were among those whose positions were abolished due to redundancy.
They received individual notices of termination and except for Verayo and Tormo, they all
signed sworn statements of conformity to the company retrenchment program. Private
respondents then filed complaints for illegal dismissal, claiming that they were singled out for
separation by reasons of their active participation in the union. They likewise claim that Asian
Alcohol was not bankrupt as it has engaged in an aggressive scheme of contractual hiring. 

VALID DISMISSAL. The requirements for valid retrenchment which must be proved by clear
and convincing evidence are: (1) that the retrenchment is reasonably necessary and likely to
prevent business losses which, if already incurred, are not merely de minimis, but substantial,
serious, actual and real, or if only expected, are reasonably imminent as perceived objectively
and in good faith by the employer; (2) that the employer served written notice both to the
employees and to the Department of Labor and Employment at least one month prior to the
intend date of retrenchment; (3) that the employer pays the retrenched employees separation
pay equivalent to one month pay or at least 1/2 month pay for every year of service, whichever
is higher; (4) that the employer exercises its prerogative to retrench employees in good faith for
the advancement of its interest of its interest and not to defeat or circumvent the employees'
right to security of tenure; and (5) that the employer used fair and reasonable criteria in
ascertaining who would be dismissed and who would be retained among the employees, such
as status, efficiency, seniority, physical fitness, age, and financial hardship for certain workers. 

An employer's good faith in implementing a redundancy program is not necessarily destroyed


by availment of the services of an independent contractor to replace the services of the
terminated employees. In the case at bar, private respondents failed to proffer any proof that the
management acted in a malicious or arbitrary manner in engaging the services of an
independent contractor.
FLIGHT ATTENDANTS AND STEWARDS ASSOCIATION OF THE PHILIPPINES (FASAP) V.
PHILIPPINE AIRLINES, INC.
(AFS not absolute requirement for valid retrenchment; judicial notice of company undergoing
corporate rehabilitation)

Background: Petitioner FASAP is the duly certified collective bargaining representative of PAL
cabin crew personnel. On June 15, 1998, PAL retrenched 5,000 of its employees, including
more than 1,400 of its cabin crew personnel. PAL adopted its so-called Plan-14 retrenchment
scheme allegedly to cut costs and mitigate huge financial losses brought about by the Asian
financial crisis. Instead of basing the retrenchment on the employee’s efficiency rating and
seniority as provided by their CBA, PAL determined the cabin crew personnel efficiency ratings
through an evaluation employee's overall performance for the year 1997 alone. On July 15,
1998, PAL carried out the retrenchment of its more than 1,400 cabin crew personnel. On
September 23, 1998, PAL ceased its operations and sent notices of termination to its
employees. PAL resumed domestic and international operations after having an agreement with
FASAP and began recalling to service those it had previously retrenched.

On June 22, 1998, FASAP filed a Complaint against PAL and Chiong for unfair labor practice,
illegal retrenchment with claims for reinstatement and payment of salaries, allowances and
backwages of affected FASAP members, actual, moral and exemplary damages with a prayer
to enjoin the retrenchment program then being implemented. PAL filed for a motion to dismiss.

Facts: On 22 July 2008, the SC ruled that PAL's retrenchment program is illegal and that PAL
failed to: (1) prove its financial losses because it did not submit its audited financial statements
as evidence; (2) observe good faith in implementing the retrenchment program; and (3) apply a
fair and reasonable criteria in selecting who would be terminated. Hence, SC ordered the
reinstatement of the retrenched and demoted cabin crew.

PAL laments the unfair and unjust conclusion reached in the July 22, 2008 decision to the effect
that it had not proved its financial losses due to its non-submission of audited financial
statements. It points out that the matter of financial losses had not been raised as an issue
before the Labor Arbiter, the NLRC, the CA, and even in the petition in G.R. No. 178083 in view
of FASAP's admission of PAL having sustained serious losses; and that PAL's having been
placed under rehabilitation sufficiently indicated the financial distress that it was suffering.

Not satisfied, PAL filed the Motion for Reconsideration of the Resolution of October 2, 2009 and
Second Motion for Reconsideration of the Decision of July 22, 2008.

On September 7, 2011, the Second Division denied with finality PAL's Second Motion for
Reconsideration of the Decision of July 22, 2008. However, on October 4, 2011, the Court En
Banc issued a resolution (a) assuming jurisdiction over G.R. No. 178083; (b) recalling the
September 7, 2011 resolution of the Second Division; and (c) ordering the re-raffle of G.R. No.
178083 to a new Member-in-Charge.

This last resolution impelled FASAP to file the Motion for Reconsideration [Re: The Honorable
Court's Resolution dated 13 March 2012], praying that the September 7, 2011 resolution in G.R.
No. 178083 be reinstated.

WON PAL implemented a valid retrenchment program?


YES. Upon a critical review of the records, the SC was convinced that PAL had met all the
standards in effecting a valid retrenchment. PAL's serious financial losses were duly
established.
In determining the validity of a retrenchment, judicial notice may be taken of the financial
losses incurred by an employer undergoing corporate rehabilitation. In such a case, the
presentation of audited financial statements may not be necessary to establish that the
employer is suffering from severe financial losses.

Indeed, that a company undergoes rehabilitation sufficiently indicates its fragile financial
condition. The Court cannot be blind and indifferent to current events affecting the society and
the country's economy, but must take them into serious consideration in its adjudication of
pending cases. In that regard, Section 2, Rule 129 of the Rules of Court recognizes that the
courts have discretionary authority to take judicial notice of matters that are of public knowledge,
or are capable of unquestionable demonstration, or ought to be known to judges because of
their judicial functions. The principle is based on convenience and expediency in securing and
introducing evidence on matters that are not ordinarily capable of dispute and are not bona fide
disputed. LA, NLRC and CA all recognized the grave financial distress of PAL during the Asian
financial crisis, but this was not the case for the Special Third Division of SC.

We emphasize, too, that the presentation of the audited financial statements should not be
the sole means by which to establish the employer's serious financial losses. The
presentation of audited financial statements, although convenient in proving the
unilateral claim of financial losses, is not required for all cases of retrenchment. The
evidence required for each case of retrenchment really depends on the particular
circumstances obtaining. While financial statements for previous years may be material in
establishing the financial trend for an employer, these are not indispensable in all cases of
retrenchment. The evidence required for each case of retrenchment will still depend on its
particular circumstances. In fact, in Revidad v. National Labor Relations Commission, the
Court declared that "proof of actual financial losses incurred by the company is not a
condition sine qua non for retrenchment," and retrenchment may be undertaken by the
employer to prevent even future losses.

FE SEBUGUERO ET AL. V. NLRC, GTI SPORTSWEAR AND/OR YUJUICO


G.R. No. 115394, 27 September 1995 (Temporary Lay-off and Art. 301 by analogous
application)

The petitioners were among the 38 regular employees of GTI Sportswear Corp. (manufacturer
and exporter of ready-to-wear garments). They were given "temporary lay-off" notices by GTI on
22 January 1991 due to alleged lack of work and heavy losses caused by the cancellation of
orders from abroad and by the garments embargo of 1990. Believing that their "temporary lay-
off" was a ploy to dismiss them because of their union activities and was in violation of their right
to security of tenure, petitioners filed with NCR LA complaints for illegal dismissal, unfair labor
practice, underpayment of wages, and non-payment of overtime pay and 13th month pay.

Private respondent GTI denied the claim of illegal dismissal and asserted that it was its
prerogative to lay-off its employees temporarily for a period not exceeding 6 months to prevent
losses due to lack of work or job orders from abroad, and that the lay-off affected both union
and non-union members. It justified its failure to recall the 38 laid-off employees after the lapse
of six months because of the subsequent cancellation of job order made by its foreign
principals, which was communicated to the petitioners and the other complainants who were all
offered severance pay. 22 of the 38 complainants accepted the separation pay. The petitioners
herein did not.

LA: held that there is valid temporary lay-off but GTI is liable for constructive dismissal for failing
to recall or send notice of retrenchment to the complainants after the lapse of 6 months.
NLRC: concurred with the findings of the LA that there was a valid lay-off of the petitioners due
to lack of work, but disagreed with the latter's ruling granting back wages after 22 July 1991.

WON the NLRC acted without or in excess of jurisdiction or grave abuse of discretion
when it ruled that there was a valid and legal reduction of business and in sustaining the
theory of redundancy in justifying the dismissal of the petitioners?

NO. The petitioners' contention is based on a wrong premise or on a miscomprehension of the


statement of the NLRC. What the NLRC sustained and affirmed is not redundancy, but
retrenchment as a ground for termination of employment. They are not synonymous but distinct
and separate grounds under Article 283 of the Labor Code, as amended.
Redundancy exists where the services of an employee are in excess of what is reasonably
demanded by the actual requirements of the enterprise. A position is redundant where it is
superfluous, and superfluity of a position or positions may be the outcome of a number of
factors, such as over-hiring of workers, decreased volume of business, or dropping of a
particular product line or service activity previously manufactured or undertaken by the
enterprise.

Retrenchment, on the other hand, is used interchangeably with the term "lay-off." It is the
termination of employment initiated by the employer through no fault of the employee's and
without prejudice to the latter, resorted to by management during periods of business recession,
industrial depression, or seasonal fluctuations, or during lulls occasioned by lack of orders,
shortage of materials, conversion of the plant for a new production program or the introduction
of new methods or more efficient machinery, or of automation. Simply put, it is an act of the
employer of dismissing employees because of losses in the operation of a business, lack
of work, and considerable reduction on the volume of his business, a right consistently
recognized and affirmed by this Court.

Article 283 of the Labor Code which covers retrenchment speaks of a permanent
retrenchment as opposed to a temporary lay-off as is the case here. There is no specific
provision of law which treats of a temporary retrenchment or lay-off and provides for the
requisites in effecting it or a period or duration therefor . These employees cannot forever
be temporarily laid-off. To remedy this situation or fill the hiatus, Article 286 may be
applied but only by analogy to set a specific period that employees may remain
temporarily laid-off or in floating status . Six months is the period set by law that the
operation of a business or undertaking may be suspended thereby suspending the
employment of the employees concerned. The temporary lay-off wherein the employees
likewise cease to work should also not last longer than six months. After six months, the
employees should either be recalled to work or permanently retrenched following the
requirements of the law, and that failing to comply with this would be tantamount to
dismissing the employees and the employer would thus be liable for such dismissal.

To determine, therefore, whether the petitioners were validly retrenched or were illegally
dismissed, we must determine whether there was compliance with the law regarding a valid
retrenchment at any time within the six month-period that they were temporarily laid-off. Under
the afore-quoted Article 283 of the Labor Code, there are three basic requisites for a valid
retrenchment: (1) the retrenchment is necessary to prevent losses and such losses are proven;
(2) written notice to the employees and to the DOLE at least one month prior to the intended
date of retrenchment; and (3) payment of separation pay equivalent to one month pay or at least
½ month pay for every year of service, whichever is higher.

The law requires two notices — one to the employee/s concerned and another to the DOLE —
not just one. The notice to the DOLE is essential because the right to retrench is not an absolute
prerogative of an employer but is subject to the requirement of law that retrenchment be done to
prevent losses. The DOLE is the agency that will determine whether the planned retrenchment
is justified and adequately supported by facts.

The lack of written notice to the petitioners and to the DOLE does not, however, make the
petitioners' retrenchment illegal such that they are entitled to the payment of backwages and
separation pay in lieu of reinstatement as they contend. Their retrenchment, for not having been
effected with the required notices, is merely defective.

SANGWOO PHILIPPINES V. SANGWOO PHILIPPINES EMPLOYEES UNION


(Closure due to serious business losses no separation pay, posting of notices not compliant
with procedural due process, reduced nominal damages)

On July 25, 2003, Sangwoo Phils. Inc. (SPI) filed with the DOLE a letter-notice of temporary
suspension of operations for 1 month, beginning September 15, 2003, due to lack of orders
from its buyers. On September 15, 2003, SPI temporarily ceased operations. Thereafter, it
successively filed 2 letters with the DOLE, copy furnished SPEU (Sangwoo Union), for the
extension of the temporary shutdown until March 15, 2004.
On October 28, 2003, SPEU filed a complaint for unfair labor practice, illegal closure, illegal
dismissal, damages and attorney's fees before the Regional Arbitration Branch IV of the NLRC.
Subsequently, or on February 12, 2004, SPI posted, in conspicuous places within the company
premises, notices of its permanent closure and cessation of business operations, effective
March 16, 2004, due to serious economic losses and financial reverses. The DOLE was
furnished a copy of said notice on February 13, 2004, together with a separate letter notifying it
of the company's permanent closure. SPEU was also furnished with a copy of the notice of
permanent closure.

LA: ruled in favor of SPI and ordered that SPI’s closure is due to serious business losses, it was
not mandated by law to grant separation benefits to the minority employees
NLRC: sustained LA ruling but modified payment of separation pay to members of SPEU
CA: ruled that minority employees were not entitled to separation pay considering that the
company's closure was due to serious business losses

(1) WON the minority employees are entitled to separation pay

NO. Closure of business, as an authorized cause for termination of employment, aims to


prevent further financial drain upon an employer who cannot pay anymore his employees since
business has already stopped. In such a case, the employer is generally required to give
separation benefits to its employees, unless the closure is due to serious business losses.
Article 297 of the Labor Code does not obligate an employer to pay separation benefits when
the closure is due to serious losses. To require an employer to be generous when it is no longer
in a position to do so, in our view, would be unduly oppressive, unjust, and unfair to the
employer. Ours is a system of laws, and the law in protecting the rights of the working man,
authorizes neither the oppression nor the self-destruction of the employer. The Court thus holds
that SPI is not obliged to give separation benefits to the minority employees pursuant to Article
297 of the Labor Code as interpreted in the case of Galaxie.

(2) WON SPI complied with the notice requirement of Article 297 (formerly Article 283) of
the Labor Code

NO. Article 297 of the Labor Code provides that before any employee is terminated due to
closure of business, it must give a one (1) month prior written notice to the employee and
to the DOLE. In this relation, case law instructs that it is the personal right of the employee to
be personally informed of his proposed dismissal as well as the reasons therefor; and such
requirement of notice is not a mere technicality or formality which the employer may dispense
with.

An employer's act of posting notices to this effect in conspicuous areas in the workplace
is not enough. Verily, for something as significant as the involuntary loss of one's employment,
nothing less than an individually addressed notice of dismissal supplied to each worker
is proper.

It is well to stress that while SPI had a valid ground to terminate its employees , i.e., closure of
business, its failure to comply with the proper procedure for termination renders it liable to pay
the employee nominal damages for such omission. Considering that SPI closed down its
operations due to serious business losses and that said closure appears to have been done in
good faith, the Court — similar to the case of Industrial Timber — deems it just to reduce the
amount of nominal damages to be awarded to each of the minority employees from P50,000.00
to P10,000.00.

NAVOTAS SHIPYARD VS. MONTALLANA

Montallana, et al. were employees of petitioner and were called into a meeting where Villaflor,
President of petitioner, told them: them: “Magsasara na ako ng negosyo, babayaran ko na lang
kayo ng separation pay dahil wala na akong pangsweldo sa inyo. Marami akong mga utang sa
krudo, yelo, at iba pa.” Since then, they were not allowed to report for work but Villaflor’s
promise to give them separation pay never materialized despite their persistent demands and
follow-ups. The petitioners, on the other hand, claimed that due to the “seasonal lack of fish
caught and uncollected receivables” the company suffered financial reverses. It was thus
constrained to temporarily cease operations. It reported the temporary shutdown to the DOLE-
NCR and filed an Establishment Termination Report. Respondents filed a complaint for
constructive dismissal against petitioner and Villaflor. Petitioners argument was based on Art.
283 (now 300) while that of respondents were based on Art. 286 (now 301).

WON respondents were constructively dismissed or illegally dismissed?


WON respondents were entitled to backwages and separation pay?
WON due process was observed?

1. No. The company’s employees were not illegally dismissed; rather, they lost their
employment because the company ceased operations after failing to recover from their
financial reverses. The company did not resort to retrenchment under Art. 283 of LC (now
300) but rather a temporary shutdown under Art. 286 (now 301) where the employees are
considered on floating status or whose employment is temporarily suspended. The
temporary shutdown has ripened into a closure or cessation of operations and thus became
a permanent shutdown.
2. He is entitled to separation pay but not to backwages. Since there was no illegal dismissal,
the respondents are not entitled to backwages. In the case of separation pay, the company’s
closure was due to serious financial reverses, hence it is not legablly bound to give the
separated employee separation pay. However, Villaflor told them that he would be giving
them separation pay as a consequence of the company’s closure. He should now honor his
undertaking to the respondents and grant them separation pay.

3. No. If the dismissal is by virtue of a just or authorized cause, but without due process, the
dismissed workers are entitled to an indemnity in the form of nominal damages (in this case
P10,000).

DEOFERIO VS. INTEL TECHNOLOGY PHILIPPINES

Intel employed Deoferio as product quality and reliability engineer and assigned him to US as a
validation engineer for 2 years. He was thereafter repatriated to the Philippines after being
confined for major depression with psychosis and worked as a product engineer in the
Philippines. Dr. Lee issued a psychiatric report stating that Deoferio’s psychotic symptoms are
not curable within a period of 6 months and will negatively affect his work and social relation
with his co-workers. He was dismissed and thereafter filed a complaint for illegal dismissal
against Intel and Wentling.

WON petitioner was validly terminated due to disease under Art. 284 (now 299)?
WON twin notice requirement is applicable under Art. 284 (now 299)?

1.Yes. Dr. Lee’s psychiatric report substantially proves that Deoferio was suffering from
schizophrenia, that his disease was not curable within a period of six months even with proper
medical treatment, and that his continued employment would be prejudicial to his mental health.
This conclusion is further substantiated by the unusual and bizarre acts that Deoferio committed
while at Intel’s employ. Therefore, he cannot also claim for backwages and separation pay since
his dismissal was for an authorized cause and respondents acted in good faith.

2. Yes. The Labor Code and its IRR are silent on the procedural due process required in
terminations due to disease. Despite the seeming gap in the law, Section 2, Rule 1, Book VI of
the IRR expressly states that the employee should be afforded procedural due process in all
cases of dismissals. Nominal damages of P30,000 were granted to Deoferio

UNITED COCONUT CHEMICALS (UCCI) VS. ALMORES

UCCI hiired respondent as Senior Utilities Inspector and became a member of UELO (United
Coconut Chemicals, Inc. Employees Labor Organization) until his expulsion in 1995. Due to the
expulsion, UELO formally demanded that UCCI terminate the services of respondent pursuant
to union security clause of the CBA. UCCI dismissed him. He filed a complaint for illegal
dismissal. LA dimissed his complaint but was reversed by NLRC. CA ruled that there was illegal
dismissal although petitioner is not liable for full backwages and was appealed to the SC but
was denied. Respondent moved for execution of judgment. LA Lontoc ruled that the backwages
due to respondent should be computed by excluding the benefits under the CBA. However,
NLRC reversed agan LA and ruled to include benefits granted under the CBA. CA affirmed
NLRC.
UCCI posits that in determining the respondent's backwages the prospective increases in
wages as well as the benefits provided in the CBA should be excluded; that, as a consequence,
the base figure for computing the respondent's backwages should be his basic salary prevailing
at the time of his dismissal, unqualified by deductions or increases

WON benefits provided under the CBA should be excluded?

No. Backwages includes all benefits previously enjoyed by the illegally dismissed employee.
The base figure for the computation of backwages should include not only the basic salary but
also the regular allowances being received, such as the emergency living allowances and the
13th month pay mandated by the law. The amount does not include the increases or benefits
granted during the period of his dismissal because time stood still for him at the precise moment
of his termination, and move forward only upon his reinstatement. Hence, the respondent
should only receive backwages that included the amounts being received by him at the time of
his illegal dismissal but not the benefits granted to his coemployees after his dismissal.

CBA allowances and benefits that the respondent was regularly receiving before his illegal
dismissal should be added. Article 279 of the Labor Code decrees that the backwages shall be
"inclusive of allowances, and to his other benefits or their monetary equivalent." Considering
that the law does not distinguish between the benefits granted by the employer and those
granted under the CBA, he should not be denied the latter benefits.

SESSION DELIGHTS V. COURT OF APPEALS

Adonis Flora filed an illegal dismissal case against Session Delights and Fast Foods (Session
Delights). In February 2001, the labor arbiter, Monroe Tabingan, ruled in favor of Flora and
ordered Session Delights to pay Flora.

Instead of paying Flora, Session Delights appealed the decision of the arbiter. The arbiter made
the recomputation and the petitioner was ordered now to pay Flora the amount of P253K. 

Petitioner again appealed to the CA, but this time it only appealed the recomputation as it avers
that the amount in the original decision of the Labor Arbiter should be controlling, in short,
the period that lapsed during the time of appeal should not be included in computing the
backwages. CA ruled in favor of the labor arbiter.

W/N the computation is correct.

YES. AS A RULE, BACKWAGES IS COMPUTED FROM THE TIME OF THE ILLEGAL


DISMISSAL UP TO THE TIME OF THE ACTUAL REINSTATEMENT. HOWEVER, IF
REINSTATEMENT IS NO LONGER POSSIBLE, IT IS COMPUTED UNTIL THE FINALITY OF
THE DECISION.

In this case, the decision became final when Session Delights no longer appealed the CA
decision affirming the finding of illegal dismissal against Session Delights or on July 29, 2003.
Hence, the original computation made by the labor arbiter in its February 2001 decision must be
recomputed to include the period until July 29, 2003. The labor arbiter re-computed the award to
include the separation pay and the backwages due up to the finality of the CA decision that fully
terminated the case on the merits. 

BANI RURAL BANK V. DE GUZMAN


The respondents in this case filed a complaint for illegal dismissal against the petitioner. The
NLRC ordered that respondents be reinstated with payment of backwages from the time of their
dismissal until their actual reinstatement. Such decision has become final and executory.
Computation of backwages was referred to Labor Arbiter.

Petitioners appealed the computation of the backwages with the NLRC. The NLRC additionally
awarded the payment of separation pay, in lieu of reinstatement on account of the strained
relations between the parties.

W/N the computation of backwages are correct

YES. THE COMPUTATION OF BACKWAGES DEPENDS ON THE FINAL AWARDS


ADJUDGED AS A CONSEQUENCE OF ILLEGAL DISMISSAL. AS A RULE, BACKWAGES
IS COMPUTED FROM THE TIME OF THE ILLEGAL DISMISSAL UP TO THE TIME OF THE
ACTUAL REINSTATEMENT. HOWEVER, IF REINSTATEMENT IS NO LONGER POSSIBLE,
IT IS COMPUTED UNTIL THE FINALITY OF THE DECISION.

In this case, respondents backwages can no longer be computed up to the point of


reinstatement as there is no longer any award of reinstatement to speak of. Thus, the
computation of the respondents' backwages must be from the time of the illegal dismissal from
employment until the finality of the decision ordering the payment of separation pay. The
respondents' backwages, therefore, must be computed from the time of their illegal dismissal
until January 29, 1999, the date of finality of the NLRC's July 31, 1998 Decision.

NACAR V. GALLERY FRAMES

Petitioner filed a complaint for illegal dismissal against the respondent alleging that he was
dismissed without any just cause. The Labor Arbiter found Gallery Frames guilty of illegal
dismissal hence the Arbiter awarded Nacar in damages consisting of backwages and separation
pay. Gallery Frames appealed all the way to the Supreme Court.

After the finality of the SC decision affirming the decision of the LA, Nacar filed a motion before
the LA for recomputation as he alleged that his backwages should be computed from the time of
his illegal dismissal until the finality of the SC decision with interest. The LA denied the motion
as he ruled that the reckoning point of the computation should only be from the time Nacar was
illegally dismissed until the decision of the LA.

W/N the petitioner is correct

YES. AS A RULE, BACKWAGES IS COMPUTED FROM THE TIME OF THE ILLEGAL


DISMISSAL UP TO THE TIME OF THE ACTUAL REINSTATEMENT. HOWEVER, IF
REINSTATEMENT IS NO LONGER POSSIBLE, IT IS COMPUTED UNTIL THE FINALITY OF
THE DECISION.

In other words, backwages will be computed from the date of illegal dismissal until the date of
the decision of the Labor Arbiter. But if the employer appeals, then the end date shall be
extended until the day when the appellate court’s decision shall become final. Hence, as a
consequence, the liability of the employer, if he loses on appeal, will increase – this is just but a
risk that the employer cannot avoid when it continued to seek recourses against the Labor
Arbiter’s decision. 

The recomputation of the consequences of illegal dismissal upon execution of the decision does
not constitute an alteration or amendment of the final decision being implemented. The illegal
dismissal ruling stands; only the computation of monetary consequences of this dismissal is
affected, and this is not a violation of the principle of immutability of final judgments. 

LARA’S GIFT & DECORS v. MIDTOWN INDUSTRIAL SALES

Lara’s is engaged in the manufacture, selling and exporting of handicrafts. Midtown is engaged
in selling industrial and construction materials and Lara is one of its clients. Lara made several
purchases from Midtown on a sixty (60)-day credit term, with the condition that 24% interest per
annum would be charged on all accounts overdue, as stated in the sales invoices. Midtown was
issued several checks which apparently bounced more than once, even after repeated demands
from Lara. Lara interposed that non -payment was due to the poor quality of products delivered
by Midtown, which caused its failing the standard of its US buyers, raising also the economic
recession in US and the fire incident in its factory. Lara questioned the validity of the 24%
interest rate.

THE 24% INTEREST RATE IS VALID. The general rule is that the interest stipulated by the
parties shall apply, provided it is not excessive and unconscionable. Absent any stipulation, the
prevailing legal interest prescribed by BSP applies to loans or forbearance of money, goods or
credits, as well as to judgments.

In the present case, the validity of interest rate fixed at 24% per annum that was expressly
stipulated in the sales invoices. The Court held that petitioner is presumed to have full
knowledge of the terms and conditions of the contract and that by not objecting to the
stipulations in the sales invoice, it also bound itself to pay not only the stated selling price but
also the interest of 24% per annum on overdue accounts and the 25% of the unpaid invoice for
attorney's fees.

OLYMPIA HOUSING v. PASTORA

Olympic Housing, Inc. (OHI) is engaged in the management of the Olympia Executive
Residences (OER), a condominium hotel building situated in Makati City. Defendants Felix
Limcaoco (part owner of OHI) and Fast Manpower (legitimate manpower agency of OHI).
Lapastora and Ubalubao worked as room attendants of OHI and they were found to have
violated house rules and regulations and were reprimanded accordingly. They filed for illegal
dismissal, payment of backwages and other benefits, and regularization of employment.
Petitioners alleged that they were not employees of the company but of Fast Manpower.

Thereafter, OHI informed the DOLE of its cessation of operations due to a MOA executed
stipulating the change of management of the condo-hotel from OHI to Raintree and issued
notices of termination to all its employees.

RESPONDENT LAPASTORA WAS ILLEGALLY DISMISSED. By the nature of OHI’s


business, it is imperative that it maintains a pool of housekeeping staff. The fact alone that
Lapastora was allowed to work for an unbroken period of almost five years is a reason to
consider him a regular employee. A regular status of employment guarantees the employee's
security of tenure that he cannot be unceremoniously terminated from employment.

On the substantive aspect, it appears that OHI failed to prove that Lapastora's dismissal was
grounded on a just or authorized cause. On the procedural aspect, OHI admittedly failed to
observe the twin notice rule in termination cases.
HOWEVER, REINSTATEMENT IS NOT POSSIBLE. Thus, while the finding of illegal dismissal
in favor of Lapastora subsists, his reinstatement was rendered a legal impossibility with OHI’s
closure of business because reinstatement presupposes that the previous position from which
one had been removed still exists or at least of similar nature as the one previously occupied.

Considering the impossibility of Lapastora’s reinstatement, the payment of separation pay, in


lieu thereof, is proper. The amount of separation pay to be given to Lapastora must be
computed from the time he commenced employment with OHI, until the time when the company
ceased operations. As a twin relief, Lapastora is likewise entitled to the payment of backwages,
computed from the time he was unjustly dismissed until when his reinstatement was rendered
impossible without fault on his part.

CLAUDIA’S KITCHEN v. TANGUIN

Tanguin was employed by Claudia's Kitchen, Inc. as a billing supervisor in Manila Jockey Club.
She was allegedly forcing her co-employees to buy silver jewelry from her during office hours
and inside the company premises, In order to conduct a thorough investigation, she was placed
under preventive suspension. Tanguin admitted that she was selling silver jewelry, but she
denied that she did so during office hours. During her suspension, she was barred by a security
guard from entering the company premises. Tanguin filed for illegal dismissal with money
claims.

SEPARATION PAY IN LIEU OF REINSTATEMENT CANNOT BE AWARDED TO AN


EMPLOYEE NOT DISMISSED. There was neither dismissal nor abandonment. At the time
Tanguin initiated the illegal dismissal case, the complaint had no basis. The status quo ante was
that she was being asked to explain the accusation against her. Instead of complying, she opted
to file a complaint for illegal dismissal. It was premature, if not pre-emptive, which the Court
cannot tolerate or accommodate. At this time, her plea for reinstatement, backwages and/or
separation pay cannot be granted. Respondent should return to work and answer the
complaints against her and the petitioners should accept her, without prejudice to the result of
the investigation against her.

Separation is pay warranted when the cause for termination is not attributable to the employee's
fault. As an exception, case law allows the grant of separation pay or financial assistance to a
legally-dismissed employee as a measure of social justice or on grounds of equity. The Court
allowed the grant when the employee was validly dismissed for causes other than serious
misconduct or those reflecting on his moral character. The payment of separation pay and
reinstatement are exclusive remedies. The circumstances in this case, however, does not
warrant an application of the exception. The doctrine of strained relations, upon which the CA
relied on to support its pay award of separation to Tanguin, has also no application in this case.

Further, the filing of an illegal dismissal case alone is not sufficient reason to engender a
conclusion that the relationship between employer and employee is already strained. The
doctrine on strained relations cannot be applied indiscriminately since every labor dispute
almost invariably results in strained relations; otherwise, reinstatement can never be possible
simply because some hostility is engendered between the parties as a result of their
disagreement.

PCL SHIPING V NLRC


Rusel was employed as seaman by PCL Shipping, for and on behalf of its foreign principal, U-
Ming Marine. Rusel joined the vessel M/V Cemtex for 12 months. 

While Rusel was cleaning the vessel’s kitchen, he slipped and therefore suffered a
broken/sprained ankle on his left foot. A request for medical examination was denied by its
captain. One day, feeling unbearable pain in his ankle, Rusel jumped off the vessel using a life
jacket and swam to the shore. He was brought to a hospital and was confined for 8 days. A
vessel’s agent fetched Rusel from the hospital and was required to board a plane bound for the
Philippines. Rusel filed a complaint for Illegal Dismissal, non payment of wages, OT pay, Sick
leave, medical benefits and damages against petitioner PCL Shipping and U-Ming Marine. In
their answer, the latter alleged that Rusel deserted his employment by jumping off the vessel.
LA found petitioners liable for unjust repatriation and awarded among other attorney’s fees
equivalent to 10% of the monetary award.
Petitioners argue that the award of attorney’s fees should be deleted because there is nothing in
the decision of the LA/NLRC which states the reason why attorney’s fees are being awarded.

WON Rusel is entitled to the award of attorney’s fees?

YES. There are two commonly accepted concepts of attorney's fees, the so-called ordinary and
extraordinary. In its ordinary concept, an attorney's fee is the reasonable compensation paid to
a lawyer by his client for the legal services he has rendered to the latter. The basis of this
compensation is the fact of his employment by and his agreement with the client. In its
extraordinary concept, attorney's fees are deemed indemnity for damages ordered by the
court to be paid by the losing party in a litigation. The instances where these may be
awarded are those enumerated in Article 2208 of the Civil Code, specifically par. 7 thereof
which pertains to actions for recovery of wages, and is payable not to the lawyer but to the
client, unless they have agreed that the award shall pertain to the lawyer as additional
compensation or as part thereof. The extraordinary concept of attorney's fees is the one
contemplated in Article 111 of the Labor Code, which provides:

Art. 111. Attorney's fees. – (a) In cases of unlawful withholding of wages, the culpable party may
be assessed attorney's fees equivalent to ten percent of the amount of wages recovered x x x

The afore-quoted Article 111 is an exception to the declared policy of strict construction in the
awarding of attorney's fees. Although an express finding of facts and law is still necessary to
prove the merit of the award, there need not be any showing that the employer acted
maliciously or in bad faith when it withheld the wages. There need only be a showing that the
lawful wages were not paid accordingly, as in this case.

In the present case, it is true that the Labor Arbiter and the NLRC failed to state the reasons
why attorney's fees are being awarded. However, it is clear that the private respondent was
illegally terminated from his employment and that his wages and other benefits were withheld
from him without any valid and legal basis. As a consequence, he is compelled to file an action
for the recovery of his lawful wages and other benefits and, in the process, incurred expenses.
On these bases, the Court finds that he is entitled to attorney's fees.

PADILLO V RURAL BANK OF NABUNTURAN

Petitioner Padillo was an employee of respondent rural bank as its SA Bookkeeper. Due to
liquidity problems in 2003, the bank took out retirement/insurance plans with Philam Life for all
its employees in anticipation of its possible closure and the concomitant severance of its
personnel. 
Padillo suffered a mild stroke which impaired his ability to effectively pursue his work. He wrote
Oropeza (President and Major Stockholder of the bank) of his intent to avail of an early
retirement package. His request remained unheeded. Not having received his claimed
retirement benefits, Padillo filed with the NLRC a complaint for recovery of unpaid retirement
benefits.
LA dismissed the complaint on the ground that Padillo did not qualify to receive any benefits
under Art 300 of LC as he was only 55 years old when he resigned, while the law specifically
provides for an optional retirement age of 60 and compulsory retirement age of 65. However, LA
directed the bank to pay Padillo P100,000 as financial assistance, treated as an advance from
the amounts receivable under Philam Life.

WON Padillo is entitled to retirement benefits?

No. Art 300 of LC: Simply stated, in the absence of any applicable agreement, an employee
must (1) retire when he is at least sixty (60) years of age and (2) serve at least (5) years in the
company to entitle him/her to a retirement benefit of at least one-half (1/2) month salary for
every year of service, with a fraction of at least six (6) months being considered as one whole
year.

All told, in the absence of any applicable contract or any evolved company policy, Padillo should
have met the age and tenure requirements set forth under Article 300 of the Labor Code to be
entitled to the retirement benefits provided therein. Unfortunately, while Padillo was able to
comply with the five (5) year tenure requirement as he served for twenty-nine (29) years he,
however, fell short with respect to the sixty (60) year age requirement given that he was only
fifty-five (55) years old when he retired. Therefore, without prejudice to the proceeds due under
the Philam Life Plan, petitioners claim for retirement benefits must be denied.

Nevertheless, the Court concurs with the CA that financial assistance should be awarded but at
an increased amount. With a veritable understanding that the award of financial assistance is
usually the final refuge of the laborer, considering as well the supervening length of time which
had sadly overtaken the point of Padillos death an employee who had devoted twenty-nine (29)
years of dedicated service to the Bank the Court, in light of the dictates of social justice, holds
that the CAs financial assistance award should be increased from P50,000.00 to P75,000.00,
still exclusive of the P100,000.00 benefit receivable by the petitioners under the Philam Life
Plan which remains undisputed.

DE LA SALLE UNIVERSITY ARANETA V BERNARDO

Bernardo is a part time professional lecturer at petitioner, DLS-AU. He taught for 2 semesters
and a summer school year. He took a leave of absence when he was assigned by the Philippine
Government to work at Papua New Guinea. When Bernardo came back, he resumed teaching
at petitioner DLS-AU. However,he was informed that he could not teach at the school anymore
as the school was implementing the retirement age limit for its faculty members as he was
already 75, Bernardo had no choice but to retire.
Bernardo sought advice from DOLE regarding his entitlement to retirement benefits after 27
years of employment. DOLE said that Bernardo was entitled to receive benefits under the New
Retirement Law (RA 7641).
Dr.  Bautista (manager of Petitioner school) told Bernardo that he was not entitled to any kind of
separation pay or benefits. That as mandated by the DLS-AU's policy and CBA, only full-time
permanent faculty of DLS-AU for at least five years immediately preceding the termination of
their employment could avail themselves of the postemployment benefits. As a part-time faculty
member, Bernardo did not acquire permanent employment under the Manual of Regulations for
Private Schools, in relation to the Labor Code, regardless of his length of service.

Bernardo filed before the NLRC a complaint for non payment of retirement benefits and
damages against DLS-AU and Dr. Bautista.

WON part-time employees are excluded from the coverage of those entitled to retirement
benefits under  RA 7641/New Retirement Law?

No. Republic Act No. 7641 states that "any employee may be retired upon reaching the
retirement age x x x;" and "[i]n case of retirement, the employee shall be entitled to receive such
retirement benefits as he may have earned under existing laws and any collective bargaining
agreement and other agreements." The Implementing Rules provide that Republic Act No.
7641 applies to "all employees in the private sector, regardless of their position,
designation or status and irrespective of the method by which their wages are paid,
except to those specifically exempted x x x." And Secretary Quisumbing' s Labor Advisory
further clarifies that the employees covered by Republic Act No. 7641 shall "include part-
time employees, employees of service and other job contractors and domestic helpers or
persons in the personal service of another."

The only exemptions specifically identified by Republic Act No. 7641 and its IRR are: (1)
employees of the National Government and its political subdivisions, including government-
owned and/or controlled corporations, if they are covered by the Civil Service Law and its
regulations; and (2) employees of retail, service and agricultural establishments or operations
regularly employing not more than 10 employees.

Based on Republic Act No. 7641, its IRR, and Secretary Quisumbing's Labor Advisory,
Bernardo, as a part-time employee of DLS-AU, is entitled to retirement benefits. The
general coverage of Republic Act No. 7641 is broad enough to encompass all private
sector employees, and part-time employees are not among those specifically exempted
from the law. The provisions of Republic Act No. 7641 and its Implementing Rules are plain,
direct, unambiguous, and need no further elucidation. Any doubt is dispelled by the unequivocal
statement in Secretary Quisumbing's Labor Advisory that Republic Act No. 7641 applies to even
part-time employees. Under the rule of statutory construction of expressio unius est exclusio
alterius, Bernardo's claim for retirement benefits cannot be denied on the ground that he was a
part-time employee as part-time employees are not among those specifically exempted under
Republic Act No. 7641 or its Implementing Rules.

You might also like